<<

30th March 2019 Live Test conducted by Vajiram & Ravi

Based on September and October 2018 Current Affairs

Q1. Which one of the following is a purpose of ‗Swadher Greh‘ a scheme of the Government? a) It aims to provide support and rehabilitation for women in difficult circumstances. b) It aims to address the issue of declining Child Sex Ratio (CSR). c) It aims to improve the nutritional and health status of women and children. d) It provides skill development training to women in women friendly trades.

Answer: A

Explanation: Recently, the Union Minister for Women & Child Development inaugurated the widows‘ home ‗Krishna Kutir‘ at Vrindavan, Mathura, . It is a special home for 1000 widows set up under Swadhar scheme. The Swadhar scheme was launched by the Union Ministry of Women and Child Development for rehabilitation of women in difficult circumstances. The scheme envisions a supportive institutional framework for women victims of difficult circumstances so that they could lead their life with dignity and conviction. It envisages that shelter, food, clothing, and health as well as economic and social security are assured for such women. There are more than 300 Swadhar Homes across the country under the scheme. Under it, funds are released to the implementing agencies (which are mainly NGOs). Hence, option a) is the correct answer.

Source: http://vajiramias.com/current-affairs/swadhar- scheme/5c17da172099372ce742329f/

Q2. Which of the following countries are the members of the Initiative for Multi-Sectoral Technical and Economic Cooperation (BIMSTEC)?

(1)

(2) Bangladesh

(3) Sri Lanka

(4) Thailand

(5) Malaysia

Select the correct answer using codes given below: a) 1, 2 and 3 only b) 2, 4 and 5 only c) 1, 2, 3 and 4 only d) 2, 3, 4 and 5 only

Answer: c

Explanation:

The Bay of Bengal Initiative for Multi-Sectoral Technical and Economic Cooperation (BIMSTEC) is a regional organization comprising of seven member states in South Asia and Southeast Asia lying in littoral and adjacent areas of Bay of Bengal constituting a contiguous regional unity. Technological and economic cooperation among South Asian and Southeast Asian countries along the coast of the Bay of Bengal is its main objective. This sub-regional organisation came into being on June 6, 1997, through the Bangkok Declaration. It is headquartered in Dhaka, Bangladesh. It comprises of seven member countries: five deriving from South Asia -- including Bangladesh, Bhutan, India, Nepal, Sri Lanka -- and two from Southeast Asia, including Myanmar and Thailand. Hence, option c) is the correct answer.

The Fourth BIMSTEC Summit concluded in Kathmandu (September 2018) after issuing an 18-point Kathmandu Declaration. The Kathmandu Declaration underlined the importance of multidimensional connectivity as a key enabler to economic integration of the region. The member states also signed a Memorandum of Understanding (MoU) for the establishment of a BIMSTEC Gird Interconnection to enhance energy cooperation among the member states.

Source: http://vajiramias.com/current-affairs/bimstec/5c17d8fc2099372ce5792fc8/

Q3. With reference to ‗Red Sanders‘, sometimes seen in the news, consider the following statements:

(1) It has a highly restrictive distribution in the north eastern portion of the Indian peninsula to which it is endemic.

(2) It is one of the most important trees in the tropical rain forest areas of north eastern India.

(3) It is listed as Near Threatened on IUCN Red list of Threatened Species.

Which of the statements given above are not correct? a) 1 and 2 only b) 2 and 3 only c) 1 and 3 only d) 1, 2 and 3

Answer: a

Explanation:

Recently Red sanders Anti-Smuggling Task Force (RSASTF) killed a red sanders smuggler and seized a lorry with two tonnes of red sanders logs worth over ₹1 crore. Red sanders smuggling is rampant due to its huge demand in China,

and South-East Asia for medicine, dye-making and manufacturing furniture, musical instruments, chess-sets. Operation Sesha has been launched by directorate of revenue intelligence (DRI) by involving 17 countries.

Statement (1) is incorrect-Red Sanders has a highly restrictive distribution in the South Eastern portion of Indian peninsula to which it is endemic. The Palakonda and Seshachalam hill ranges of Cuddapah-Chittoor districts of the State of Andhra Pradesh are its principal geographical range which extends slightly into the neighbouring Anantapur, Kurnool, Prakasam and Nellore Districts of Andhra Pradesh. Sporadic wild populations occur in the adjoining districts of the neighbouring states of & Karnataka.

Statement (2) is incorrect- Red Sanders bearing forest is a one of the climax forest type in tropical dry deciduous forests in India. The natural habitat of Red Sanders is a hilly region with hot dry climate.

Statement (3) is correct- It is listed as Near Threatened on IUCN Red list of Threatened Species. It is internationally protected under CITES (convention on international trade in endangered species) Appendix II since 1995.

Source: http://vajiramias.com/current-affairs/red-sanders/5c184eee2099372ce5792fd7/

Q4. Consider the following statements about the Brow antlered deer (Sangai):

(1) It is found only in Keibul Lamjao National Park.

(2) It is listed as vulnerable on IUCN Red List of Threatened Species and Schedule 1 of Wildlife (Protection) Act, 1972.

Which of the statements given above is/are correct? a) 1 only b) 2 only c) Both 1 and 2 d) Neither 1 nor 2

Answer: a

Explanation:

Activists are opposing the government‘s proposal of shifting some Brow-antlered deer (the Sangai) species – found only in ‘s Bishnupur district – to other areas. The Sangai is the state animal of Manipur. It is also known as the Dancing Deer.

Sangai is a medium-sized deer, with uniquely distinctive antlers, with extremely long brow tine, which form the main beam. The forward protruding beam appears to come out from the eyebrow, hence called the brow-antlered deer. It has a dark reddish- brown winter coat, which turns paler in summer. The deer walks on the hind surface of its pasterns with mincing hops over floating foliage, and is hence also called the Dancing Deer.

Statement (1) is correct- It is found only in Keibul Lamjao National Park. The Keibul Lamjao National Park is a national park in the Bishnupur district of the state of Manipur in India. It is the only floating park in the world, and is an integral part of Loktak . The national park is characterized by many floating decomposed plant materials locally called phumdis.

Statement (2) is incorrect- It is listed as endangered on IUCN Red List of Threatened Species and Schedule 1 of Wildlife (Protection) Act, 1972.

Sangai faces threat from steadily degenerating habitat of phumdi as a result of continuous inundation and flooding caused due to artificial reservoir. There is also invasion of non-native plants like Paragrass. It also faces threats of diseases from the livestock, inbreeding depression and .

Source: http://vajiramias.com/current-affairs/dancing- deer/5c184e9d2099372ce74232ae/

Q5. In the context of the ‗Indian Post Payments Bank (IPPB)‘, often mentioned in the news, consider the following statements:

(1) The IPPB is not authorized to advance loans or issue credit cards or offer remittance services.

(2) The maximum limit on deposits in the IPPB is Rs 1 lakh, beyond which the account will be automatically converted into post office savings account.

Which of the statements given above is/are correct? a) 1 only b) 2 only c) Both 1 and 2 d) Neither 1 nor 2 Answer: b

Explanation:

India Post Payments Bank (IPPB) has been setup under the Department of Posts, Ministry of Communication with 100% equity owned by . By leveraging the vast network of the Department of Posts (with more than 3 lakh Postmen and Grameen Dak Sewaks), IPPB will significantly augment the reach of the banking sector in the country especially in the rural areas.

Statement (1) is incorrect- The operations of IPPB will be on a smaller scale as compared to other banks and will not advance loans or issue credit cards to avoid risk. Performing the rest of its functions, it will accept deposits, offer remittance services, mobile payments/transfers/purchases and other banking services like ATM/debit cards, net banking and third-party fund transfers. It will also deliver the benefits of schemes such as Pradhan Mantri Fasal Bima Yojana.

Statement (2) is correct- The maximum limit on deposits is Rs 1 lakh, beyond which the account will be automatically converted into post office savings account. The Department of Posts (DoP) wants to ensure that a customer‘s cash balance that exceeds ₹1 lakh in its payments bank remains within the postal system through a sweep-out mechanism, whereby the excess balance will be channelised to their Post Office Savings Bank (POSB) account at the end of the day.

Source: http://vajiramias.com/current-affairs/india-post-payments-bank- ippb/5c184e5e2099372ce5792fd5/

Q6. The term ―monopsony‖ was in the news recently. Which one of the following statements provides its most apt description? a) It is a market condition where a large seller, controls a sizeable proportion of the market. b) It is a market condition where a large buyer, controls a sizeable proportion of the market. c) It is a market structure consists of a single seller and a single buyer. d) It is a formal agreement between a group of producers to regulate supply in an effort to regulate or manipulate prices.

Answer: b

Explanation:

New words such as ‗monopsony‘ and the ‗Amazon effect‘ have been coined by US central bankers and economists in the background of growing dominance of few companies and its impact on wages, inflation and growth.

A monopsony, also called a buyer‘s monopoly, is a market condition similar to a monopoly but where a large buyer, and not a seller, controls a sizeable proportion of the market and drives prices down. There are several scenarios where a monopsony can occur. Like a monopoly, a monopsony also does not adhere to standard pricing from balancing supply-side and demand-side factors. In a monopoly, where there are few suppliers, the controlling entity can sell its product at a price of its choosing because buyers are willing to pay its designated price. In a monopsony, the controlling body is a buyer. This buyer may use its size advantage to obtain low prices because many sellers vie for its business. Monopsonies commonly experience low prices from wholesalers and an advantage in paid wages. Hence, option b) is the correct answer.

Amazon effect has been coined to describe fast-changing pricing algorithms by the online retailer and its rivals, which could potentially lead to bigger swings in inflation.

Source: http://vajiramias.com/current-affairs/monopsony-amazon- effect/5c1851312099372ce74232b6/

Q7. Recently the United Nations has initiated talks to regulate the high seas with the goal of protecting marine biodiversity. In this context, consider the following statements:

(1) High Seas cover all parts of the mass of saltwater surrounding the globe including the Exclusive Economic Zone (EEZ) in the territorial sea.

(2) In 1982, the UN adopted the Convention on the Law of the Sea, but left the high seas free from restrictions.

Which of the statements given above is/are correct? a) 1 only b) 2 only c) Both 1 and 2 d) Neither 1 nor 2 Answer: b

Explanation:

Recently, United Nations has initiated talks, which will take place over two years, on a 2020 treaty that would regulate the high seas with the goal of protecting marine biodiversity. High seas cover 46% of the planet‘s surface, yet lack adequate

environmental protection. That‘s why United Nations has initiated talks for the new treaty.

Statement (1) is incorrect- High seas, in maritime law, means all parts of the mass of saltwater surrounding the globe that are not included in the Exclusive Economic Zone (EEZ), in the territorial sea or in the internal waters of a state, or in the archipelagic waters of an archipelagic state.

Statement (2) is correct- In 1982, the UN adopted the Convention on the Law of the Sea, but left the high seas free from restrictions. The convention took effect in 1994, without the participation of the U.S. Since then, shipping routes have expanded considerably, and the resources of the ocean deep have aroused significant interest, whether by fishing or mineral extraction.

Source: http://vajiramias.com/current-affairs/high-seas/5c1850932099372ce8f4db88/

Q8. Which of the following pairs are correctly matched?

Tradition States

1. Mongmong festival

2. Hojagiri folk dance

3. Mardani Khel

Select the correct answer using the code given below: a) 1 and 2 only b) 1 and 3 only c) 2 and 3 only d) 1, 2 and 3

Answer: a

Explanation:

Pair (1) is correctly matched: Recently, Mongmong, a harvest festival, was celebrated by Sangtam Naga Tribe hailing from the Tuensang/Kiphire districts of Nagaland. Mongmong in Sangtam dialect literally means ―togetherness forever‖. It is annually celebrated for six days during the first week of September as a festival of togetherness, forgiveness and prayer for a bountiful harvest. Main reasons to celebrate the festival is to worship their God and three cooking stones in the fire place.

Pair (2) is correctly matched: Hojagiri is a folk dance, performed in the state of Tripura, India by the Bru/ people. It is performed by women and young girls, about 4 to 6 members in a team, singing, balancing on an earthen pitcher and managing other props such as a bottle on the head and earthen lamp on the hand. While only the lower half of the body is moved. The dance is performed on the occasion of Hojagiri festivals or Laxmi Puja, held in the following full moon night of Durga Puja, generally after 3rd day of Dashera. The Goddess Mailuma, (Laxmi) is worshipped on this day. 'Buisu', not 'bihu' is the most popular festival of Reang tribes. They speak the Reang dialect of Kokborok language which is locally referred to as Kau Bru. The Brus, also referred to as the , are spread across the north-eastern states of Tripura, Assam, Manipur, and . Recently Eleven more families of displaced Brus are set to return to Mizoram from relief camps in adjoining Tripura.

Pair (3) is not correctly matched: Originally from , Mardani Khel is a weapon-based martial art form. It owes its development to the geographic conditions of the state (hills, caves and valleys). A very ancient form of art, it saw its emergence during the Maratha dynasty. Kolhapur used to be the centre of the Maratha kingdom and the villages around it had talims (training centres) where skilled elders prepared youngsters for war. After the revolt of 1857, the British banned the use of weapons and the talims were forced to turnmardani khel into a folk game to ensure its survival. The use of weapons such as swords, katyar (dagger), lathi-kathi (bamboo sticks), veeta (darts), bhala (javelin), dand and patta (long-bladed swords) continued but the moves were made more stylised and less lethal.

Source: http://vajiramias.com/current-affairs/mongmong/5c1853782099372ce8f4db96/ http://vajiramias.com/current-affairs/bru-refugees/5c1902fb2099377308fff98a/

Q9. The term ‗Staphylococcus epidermidis‘ which was recently seen in the news is related to which of the following? a) Viral hemorrhagic fever b) Multidrug-resistant bug c) Skin infection caused by a fungus d) Parasitic diseases caused by Protozoa.

Answer: b

Explanation:

Recently, Scientists in Australia warned a superbug resistant to all known antibiotics that can cause ―severe‖ infections or even death is spreading undetected through hospital wards across the world. Staphylococcus epidermidis is a bacterium related to the better-known and deadlier MRSA superbug ("Superbugs" is a term used to describe strains of that are resistant to the majority of antibiotics commonly used today). It‘s found naturally on human skin. Patients with compromised immune systems are at risk of developing infection. It commonly infects the elderly or patients who have had prosthetic materials implanted, such as catheters and joint replacements. It can cause ‗severe‘ infections or even death. Hence, option b) is the correct answer.

Source: http://vajiramias.com/current-affairs/staphylococcus- epidermidis/5c1854022099372ce74232c5/

Q10. What is ―VC 11184‖, which was in news recently? a) Nuclear-powered aircraft carrier b) Torpedo launch and recovery vessel c) Missile tracking ship d) Nuclear ballistic missile submarine

Answer: c

Explanation:

Hindustan Shipyard Limited (HSL) is planning to undertake sea trials of India‘s first missile tracking ship. It has been built by Hindustan Shipyard Limited (HSL) for the National Technical Research Organisation, the technical intelligence agency working directly under the supervision of the Prime Minister‘s Office and the National Security Adviser. It is a first of its kind ocean surveillance ship being built as part of the efforts to strengthen the country‘s strategic weapons programme. The ship has the capacity to carry 300-strong crew with hi-tech gadgets and communication equipment and a large deck capable of helicopter landing. It will be named after its induction into the Indian Navy. For now, it is referred as VC 11184. Once ready, this would put India in the elite club of a few countries that have such a sophisticated ocean surveillance ship. Hence, option c) is the correct answer.

INS Astradharini is India‘s first indigenously-designed and built torpedo launch and recovery vessel.

INS Arihant is India's first nuclear ballistic missile submarine. It is armed with four K-4 submarine launched ballistic missile with a range of 3,500 km. It can also be armed with 12 K-15 missiles with a range of 750 km. It is a part of Indian Navy's secretive Advanced Technology Vessel (ATV) project.

Source: http://vajiramias.com/current-affairs/vc-11184/5c187d762099372ce7423340/

Q11. Recently, the Ministry of Shipping was proposed to set up a World Class National Maritime Heritage Complex (NMHC) at ______. a) Mumbai, Maharashtra b) Chennai, Tamil Nadu c) Lothal, d) Kakinada, Andhra Pradesh

Answer: c

Explanation:

Shipping Ministry held a consultative workshop on expediting the project for development of a World-Class National Maritime Heritage Complex (NHMC) at Lothal in Gujarat. NHMC is being developed by Ministry of Shipping under the Sagarmala programme. It will showcase and preserve India‘s rich and diverse maritime heritage. It will display objects relating to ships and travel on large bodies of water. It also intends to highlight the ancient shipbuilding and navigational technologies developed by India. Hence, option c) is the correct answer.

Lothal is one of the most important Harappan sites in Gujarat which clearly defined maritime activity those times. The dockyard — 37 metres x 22 metres in dimensions — is believed to be the era's (3700 BCE) finest engineering marvel created after studying tidal movements and creating a pathway to the world.

Source: http://vajiramias.com/current-affairs/national-maritime-heritage- complex/5c18571f2099372ce8f4db9f/

Q12. Consider the following statements with reference to the Internal Ombudsman (IO) Scheme, 2018:

(1) All scheduled commercial banks and regional rural banks with over 10 branches should appoint an internal ombudsman.

(2) As per the scheme, the Internal Ombudsman will have a fixed three-to-five-year tenure, which is not extendable, but is eligible for re-appointed.

Which of the statements given above is/are correct? a) 1 only b) 2 only c) Both 1 and 2 d) Neither 1 nor 2 Answer: d

Explanation:

The Reserve Bank of India (RBI) has introduced Internal Ombudsman Scheme, 2018 for stronger redressal of banks customer‘s complaints.

Statement (1) is incorrect- All scheduled commercial banks with over 10 branches are required to appoint an internal ombudsman (IO). The central bank has, however, excluded regional rural banks (RRBs) from appointing ombudsman.

Statement (2) is incorrect- As per the scheme, the IO will have a fixed three-to-five- year tenure, which is not extendable. Also, they cannot be re-appointed.

RBI has introduced ‗Internal Ombudsman Scheme, 2018‘ under Section 35 A of the Banking Regulation Act, 1949. The IO will examine customer complaints related to deficiency in service provided by the bank. The scheme will also include complaints listed on ground of Clause 8 of the Banking Ombudsman Scheme, 2006, that are partly or wholly rejected by the bank, RBI said. The implementation of this scheme will be

monitored by the bank‘s internal audit mechanism apart from regulatory oversight by RBI.

Source: http://vajiramias.com/current-affairs/internal- ombudsman/5c1854982099372ce5792fec/

Q13. Consider the following statements with reference to the National Nutrition Mission (Poshan Abhiyaan):

(1) Rashtriya Poshan Maah is an initiative of Ministry of Health and Family Welfare and NITI Aayog to give a push to Poshan Abhiyaan.

(2) Poshan Abhiyaan will be funded 50% by the Government of India and 50% by World Bank or other Multilateral Development Banks.

Which of the statements given above is/are correct? a) 1 only b) 2 only c) Both 1 and 2 d) Neither 1 nor 2 Answer: b

Explanation:

The Government of India has celebrated the month of September, 2018 as the National Nutrition Month (Rashtriya Poshan Maah) under the Poshan Abhiyaan.

Statement (1) is incorrect- Poshan Maah is an initiative of the Ministry of Women and Child Development and NITI Aayog to give a push to Poshan Abhiyaan. During Poshan Maah, activities like Prabhat Pheri, Poshan Melas, Nukkad Nataks, School Based Events will be held to take the message of importance of nutrition to every household. It focuses on 8 themes: (1) Antenatal Care, (2) Optimal Breastfeeding, (3) Complementary Feeding, (4) Anaemia, (5) Growth Monitoring, (6) Girls-education, diet, right age of Marriage, (7) Hygiene & Sanitation, (8) Food Fortification.

Statement (2) is correct- Poshan Abhiyaan will be funded 50% by the Government of India and 50% by World Bank or other Multilateral Development Banks. Government budgetary support would be 60:40 between Centre and States/UTs, 90:10 for North East Region and Himalayan States and 100% for UTs without legislature.

Poshan Abhiyaan (National Nutrition Mission) was launched by the Prime Minister in March, 2018 from Jhunjhunu, . By the year 2020, all 36 States/UTs and 718

districts will be covered in a phased manner. Its objective is to reduce malnutrition through use of technology, a targeted approach and convergence. Its target is to reduce stunting, under-nutrition, anaemia (among young children, women and adolescent girls) and low birth weight by 2%, 2%, 3% and 2% per annum respectively. Although the target to reduce Stunting is atleast 2% p.a., Mission would strive to achieve reduction in Stunting from 38.4% (NFHS-4) to 25% by 2022.

Source: http://vajiramias.com/current-affairs/rashtriya-poshan- maah/5c185fb82099372ce5792ff9/

Q14. Consider the following statements with reference to the Programme for International Student Assessment (PISA):

(1) It was introduced in the year 2000 by the UNESCO.

(2) It assesses the quality of education systems across the world by evaluating students in science, mathematics and reading in every three years.

Select the correct answer using the code given below: a) 1 only b) 2 only c) Both 1 and 2 d) Neither 1 nor 2 Answer: b

Explanation:

Union Human Resource Development (HRD) Ministry has decided to participate in PISA, an international assessment of student ability, in 2021 after a gap of 10 years.

Statement 1 is incorrect- Programme for International Student Assessment (PISA) was introduced in the year 2000 by the Organisation for Economic Cooperation Development (OECD).

Statement 2 is correct- It assesses the quality of education systems across the world by evaluating students in science, mathematics and reading in every three years.

The two-hour computer-based test focuses on 15-year-old students as they have either completed or are near the end of their compulsory education in most countries. PISA requires the examinees to have finished at least six years of formal schooling. About 5.5 lakh students in 72 countries took the test in 2015. However, many non-OECD members including all SAARC nations, Greenland, Argentina and the

entire African continent (except Algeria and Tunisia) either don‘t regularly participate or haven‘t participated at all in PISA. Till date, India has participated only once in PISA in 2009. India stayed away from PISA in 2012 and 2015 on account of its dismal performance in 2009, when it was placed 72nd among the 74 participating countries.

Source: http://vajiramias.com/current-affairs/pisa/5c1879712099372ce579304b/

Q15. Which of the following pairs is/are correctly matched?

Protected areas States

(1) Singphan Wildlife Sanctuary Nagaland

(2) Raiganj Wildlife Sanctuary

(3) Satkosia Tiger Reserve

Select the correct answer using the codes given below: a) 1 only b) 1 and 2 only c) 2 and 3 only d) 1, 2 and 3

Answer: b

Explanation:

Pair (1) is correctly matched - The Nagaland government has declared the Singphan Wildlife Sanctuary as an elephant reserve with the approval of Government of India in August 2018. This makes Singphan Elephant Reserve as the 30th elephant reserve in the country. It is located in Mon district of Nagaland. It is strategically located in contiguity with the Abhaypur Reserve Forest of Assam. The creation of reserve will give better protection and conservation of elephants in the state. It spreads over an area of 5825 acres (2357 hac). As per recent census report (2017), Nagaland has population of around 446 elephants.

Pair (2) is correctly matched- In a first, Raiganj Wildlife Sanctuary in west Bengal recorded nearly one lakh birds (98,532 to be precise) this year, crossing all previous records. The sanctuary is also known as the Kulik Bird Sanctuary, drawing its name

from the river Kulik. It is situated near Raiganj in Uttar Dinajpur district of West Bengal. The sanctuary is home to 164 bird species and has one of the highest numbers of Open bill stork population not only in India but in Asia which makes this sanctuary unique.

Pair (3) is not correctly matched - Satkosia Tiger Reserve comprises of two adjoining Sanctuaries of central named as Satkosia Gorge Sanctuary and Baisipalli Sanctuary. These two sanctuaries together covering an area of 963 sq.km have been notified as Satkosia Tiger Reserve in 2007. It is located in the Angul district of Odisha where the River passes through a 22 km long gorge in the Eastern Ghats mountains. The area is also a part of the Mahanadi elephant reserve. Satkosia is the meeting point of two bio-geographic regions of India; the Deccan Peninsula and the Eastern Ghats, contributing to immense biodiversity. The major plant communities are mixed deciduous forests including Sal and riverine forest.

Source: http://vajiramias.com/current-affairs/singphan-wildlife- sanctuary/5c17db182099372ce74232a4/ http://vajiramias.com/current-affairs/raiganj-wildlife- sanctuary/5c18d3ea2099377306691385/ http://vajiramias.com/current-affairs/satkosia-tiger-reserve/5c1885a32099372ce8f4dc21/

Q16. Consider the following statements with respect to the Public Credit Registry (PCR):

(1) It is an information repository that collates all loan information of individuals and corporate borrowers, across different borrowing products in one place.

(2) Vijay Kelkar committee has recommended for the setting up of a PCR which should capture all loan information and borrowers be able to access their own history.

Which of the statements given above is/are correct? a) 1 only b) 2 only c) Both 1 and 2 d) Neither 1 nor 2 Answer: a

Explanation:

RBI Deputy Governor Viral Acharya has suggested setting up of Public Credit Registry (PCR) by incorporating unique identifiers: Aadhaar for individual borrowers and Corporate Identification Number for firms.

Statement (1) is correct- A public credit registry (PCR) is an information repository that collates all loan information of individuals and corporate borrowers, across different borrowing products in one place.

Statement (2) is incorrect- A committee, headed by Y.M. Deosthalee has suggested setting of a PCR which should capture all loan information and borrowers be able to access their own history. Data is to be made available to stakeholders such as banks, on a need-to-know basis. Data privacy will be protected.

At present, credit information is now available across multiple systems and not in one window. Data on borrowings from banks, NBFCs, corporate bonds or debentures from the market, ECBs, FCCBs, masala bonds, and inter-corporate borrowings are not available in one data repository.

Benefits for India:

(i) It helps banks distinguish between a bad and a good borrower and accordingly offer attractive interest rates to good borrowers and higher interest rates to bad borrowers.

(ii) It will address issues such as information asymmetry, improve access to credit and strengthen the credit culture among consumers. (iii) It can also address the bad loan problem, as corporate debtors will not be able to borrow across banks without disclosing existing debt. (iv) It may also help to improve India‘s rankings in the World Bank‘s ease of doing business index.

Source: http://vajiramias.com/current-affairs/public-credit- registry/5c1851ae2099372ce5792fdc/

Q17. Which of the following pairs are correctly matched?

Region often mentioned in news Country

(1) Idlib Syria (2) Sulawesi Indonesia (3)

Select the correct answer using the code given below:

a) 1 and 2 only b) 1 and 3 only c) 2 and 3 only d) 1, 2 and 3

Answer: a

Explanation:

Pair (1) is correctly matched - Russia has resumed airstrikes against insurgents in the Idlib province in Syria, the last rebel bastion fighting against Syria‘s President Bashar al-Assad‘s regime. Idlib province is one of the fourteen provinces of Syria. It is situated in north-western Syria, bordering . Idlib city, the province‘s capital, lies near the road linking city Aleppo to the capital Damascus. Idlib province has been hit by several suspected chemical attacks during the ongoing civil war.

Pair (2) is correctly matched - Nearly 400 people were killed in the earthquake and the tsunami it caused, which hit the Indonesian island of Sulawesi. Sulawesi is an island in Indonesia. It is situated east of Borneo, west of the Maluku Islands, and south of Mindanao and the Sulu Archipelago. The Strait of Makassar runs along the western side of the island and separates the island from Borneo. Within Indonesia, it is smaller than Sumatra and larger than Java Island.

Pair (3) is not correctly matched- Island of Lesbos in is often in news due to refugee crisis. Lesbos is a Greek island in the northern off the coast of Turkey. It is separated from Turkey by the narrow Mytilini Strait.

Source: http://vajiramias.com/current-affairs/sulawesi/5c192f2920993773071c42b5/ http://vajiramias.com/current-affairs/idlib/5c185ec92099372ce74232dc/

Q18. IOWave18, which was recently seen in news, is related to which of the following: a) Trilateral naval exercise in involving US, Japan and India. b) Indian Ocean-wide tsunami mock exercise. c) Symposium conducted by the National Disaster Management Authority (NDMA) for Tsunami awareness and preparedness. d) International Maritime Search and Rescue Exercise in the Indian Ocean.

Answer: b

Explanation:

India, along with 23 other Indian Ocean Nations, participated in ‗IOWave18‘ – a major Indian ocean-wide tsunami mock exercise (drill) in September, 2018. IOWave18 focused on a simulating an earthquake with magnitude of 9.3 richter at Northern Sumatra, Indonesia. It was followed by issuing tsunami bulletins to national and regional stakeholders through various modes and evacuation of people in coastal regions. The Exercise was organized by the Intergovernmental Oceanographic Commission (IOC) of UNESCO, which coordinated the setting up of the Indian Ocean Tsunami Warning and Mitigation System (IOTWMS) in the aftermath of the 2004 tsunami. In India, IOWave18 was coordinated by the Indian National Centre for Ocean Information Services (INCOIS), Ministry of Earth Sciences (MoES). Hence, option b) is the correct answer.

Source: http://vajiramias.com/current-affairs/iowave18/5c1867b92099372ce5793013/

Q19. In the context of the framework for conservation of wetlands and their resources, which among the following in India are accorded the status of ‗Wetland of International Importance‘ under the Ramsar Convention?

(1) Lake

(2) Lake

(3) Islets of Ranganathittu

(4)

Select the correct answer using the code given below: a) 1 and 2 only b) 1 and 3 only c) 2, 3, and 4 only d) 1, 2, 3 and 4

Answer: a

Explanation:

Point (1) is correct- According to a report by Central Water Commission (CWC), shrinking of the carrying capacity of the Vembanad Lake in worsened the flood situation. Lake‘s capacity shrunk due to congestion; intense rain, lack of spillways and reservoirs led to surge. The situation may have been better if the capacity of the lake was larger. Vembanad Lake is also known as Vembanad Kayal, Vembanad Kol, Punnamada Lake (in Kuttanad) and Kochi Lake (in Kochi). Spanning several districts of Kerala, it is the largest lake in Kerala and the longest Lake in India. It is separated from the by a narrow barrier island and is a popular backwater stretch in Kerala. Vallam Kali (a.k.a Nehru Trophy Boat Race) is a Snake Boat Race held every year in the month of August in Vembanad Lake. In 2002 it was included in the list of wetlands of international importance, as defined by the Ramsar Convention. The Kumarakom Bird Sanctuary is located on the east coast of the lake.

Point (2) is correct- Pong Dam Lake is a reservoir created by damming of the River Beas in the foothills of the in the state of . It is also known as the . Built primarily for irrigation purpose, it became a significant habitat of migratory birds and declared as Wildlife Sanctuary in 1986. This wetland was declared as Ramsar site in 2002.

Point (3) is incorrect: The renowned islets of Ranganathittu on the banks of the river Cauvery in Srirangapatna will be pitched to be recognised as a Ramsar wetland site of international importance. There are currently 26 sites in India recognised as Ramsar wetland sites of international importance, but none of them are in Karnataka. Ranganathittu Bird Sanctuary, also known as Pakshi Kashi of Karnataka, is a bird sanctuary in the Mandya District of the state of Karnataka in India.

Point (4) is incorrect- The Gulf of Mannar is the first Marine Biosphere Reserve (GOMMBR) in the South and South East Asia, running down south from Rameswaram to Kanyakumari in Tamil Nadu. This Marine Biosphere Reserve encompasses a chain of 21 islands (2 islands already submerged) and adjoining coral reefs off the coasts of the Ramanathapuram and the Tuticorin districts forming the core zone; the Marine National Park and the buffer zone includes the surrounding seascape and a 10 km strip of the coastal landscape. Recently Tamil Nadu Government informed Madras High court that steps were underway to include the Pallikaranai marshland in Kancheepuram district, Kaluveli swamp in Villupuram and the Gulf of Mannar wetland complex under the Ramsar Convention.

Source: http://vajiramias.com/current-affairs/vembanad- lake/5c1868bc2099372ce579301b/

Q20. What of the following best describes the term ―regulatory sandbox‖, which was seen in the news recently? a) An entity hosted or endorsed by the regulators that enables temporary and limited- scale testing of a new product. b) A security mechanism in computer security for separating running programs, usually in an effort to mitigate system failures. c) A type of software testing environment that enables the combined execution of software or programs for efficient evaluation and monitoring. d) It is a broad framework for cryptocurrency regulation.

Answer: a

Explanation:

A regulatory sandbox is a framework, set up by a financial sector regulator to allow small scale, live testing of innovations by private firms in a controlled environment under the regulator's supervision.

In the digital economy arena, it provides testing grounds for new business models that are not protected by current regulation, or supervised by regulatory institutions. These testing grounds are especially relevant in the fintech world, where there is a growing need to develop regulatory frameworks for emerging business models. Regulatory sandboxes typically involve temporary relaxations or adjustments of regulatory requirements to provide a ―safe space‖ for startups or established companies to test new technology-based financial services in a live environment for a limited time, without having to undergo a full authorization and licensing process. Hence, option a) is the correct answer.

Insurance Regulatory and Development Authority of India (IRDAI) is working on the guidelines for ‗Sandbox‘ method to allow insurers to test products in a particular geography or among a set of few policyholders before they are available in the market.

Source: http://vajiramias.com/current-affairs/sandbox- method/5c186b372099372ce8f4dbd3/

Q21. Consider the following statements about the ‗Transformation of Aspirational Districts‘ programme:

(1) It is a centrally sponsored scheme launched by the Union Ministry of Rural Development.

(2) The 115 districts were identified, at least one from each state, by senior officials of the Union government in consultation with State officials based on a composite Index.

Which of the statements given above is/are correct? a) 1 only b) 2 only c) Both 1 and 2 d) Neither 1 nor 2 Answer: b

Explanation:

The ‗Transformation of Aspirational Districts‘ programme launched in January 2018 aims to quickly and effectively transform selected districts. The broad contours of the programme are Convergence (of Central & State Schemes), Collaboration (of Central, State level ‗Prabhari‘ Officers & District Collectors), and Competition among districts driven by a mass Movement. With States as the main drivers, this program will focus on the strength of each district, identify low-hanging fruits for immediate improvement, measure progress, and rank districts.

The Government is committed to raising the living standards of its citizens and ensuring inclusive growth for all. To enable optimum utilization of their potential, this program focusses closely on improving people‘s ability to participate fully in the burgeoning economy. Health & Nutrition, Education, Agriculture & Water Resources, Financial Inclusion & Skill Development, and Basic Infrastructure are this program‘s core areas of focus.

Statement (1) is incorrect- NITI Aayog anchors the Aspirational Districts programme with support from Central Ministries and the State Governments. While NITI Aayog is steering the initiative in 30 districts, various central ministries oversee 50 districts besides the Ministry of Home Affairs, which focuses on 35 Left Wing Extremism (LWE) affected districts.

Statement (2) is correct- Under the programme 115 districts were identified from 28 states, at least one from each state, in a transparent manner by a committee of Senior Officers to the Government of India, in consultation with State Officials using a composite index of key data sets that included deprivation enumerated under

the Socio-Economic Caste Census, key health and education sector performance and state of basic infrastructure.

Source: http://vajiramias.com/current-affairs/aspirational- districts/5c1872f92099372ce5793039/

Q22. As seen in news recently, Great Pacific Garbage Patch is a zone that is located between which of the following? a) Shanghai and Kyushu b) and c) Uelen and d) Sydney and Auckland

Answer: b

Explanation:

The Ocean Cleanup, a Dutch non-profit group, has initiated a project to clean up half of the infamous Great Pacific Garbage Patch (GPGP) within five years. GPGP is the largest of the five offshore plastic accumulation zones in the world‘s oceans. It was first discovered in 1997. It is also called as the Pacific Trash Vortex. It is located halfway between Hawaii and California. This zone covers an estimated surface area of 1.6 million sq km, an area twice the size of Texas or three times the size of .

The mass of the plastic in the GPGP was estimated to be approximately 80,000 tonnes. It is formed when garbage is carried into the center of an ocean gyre, there are no more outside forces acting on the garbage to move it any further. The result is that garbage will often collect at the center of ocean gyres and stay there for many years. Due to this, the North Pacific Ocean, the North Atlantic Ocean, and the Indian Ocean all have substantial patches of garbage. Hence, option b) is the correct answer.

Source: http://vajiramias.com/current-affairs/great-pacific-garbage-patch- gpgp/5c18745f2099372ce742331c/

Q23. Consider the following statements with reference to the World Summit on Accreditation (WOSA):

(1) It is a biennial Summit organised by the Ministry of Commerce and Industry.

(2) Its objective is to provide accreditation agencies from across the world a common platform to share best practices, insights and to enhance their understanding of accreditation.

Which of the statements given above is/are correct? a) 1 only b) 2 only c) Both 1 and 2 d) Neither 1 nor 2 Answer: b

Explanation:

Union Minister for Human Resources Development inaugurated the 4th World Summit on Accreditation (WOSA-2018) at New recently.

Statement (1) is incorrect- World Summit on Accreditation (WOSA) is a biennial Summit organised by National Board of Accreditation (NBA) of India.

Statement (2) is correct- Its objective is to provide Accreditation agencies from across the world a common platform to share best practices, insights and to enhance their understanding of accreditation. NBA has already organised three Summits in 2012, 2014 and 2016. WOSA 2018 is being held from 7 – 9 September, 2018 in New Delhi with the theme ―Challenges and Opportunities in outcome-based Accreditation‖.

Source: http://vajiramias.com/current-affairs/world-summit-on-accreditation-wosa- 2018/5c186e412099372ce8f4dbe0/

Q24. Consider the following fauna of India:

(1) Dugong

(2) Pondicherry shark

(3) Shark

Which of the above is/are listed as Critically Endangered on IUCN Red List of Threatened species? a) 1 only b) 1 and 3 only c) 2 and 3 only d) 1, 2 and 3

Answer: c

Explanation:

Point (2) is correct- Pondicherry shark‘ has been spotted near coastal Andhra Pradesh. This is for the third time they are spotted in the East Estuarine Ecosystem region after 2007 and 2016. Scientifically known as Carcharhinus hemiodon, it belongs to the Carcharhinidae family with a growth of 3.3 feet.

It is identified by its ‗black tips‘ of dorsal, pectoral and Tai fins. The front teeth are distinctly serrated at the base and smooth at the tip. The Pondicherry shark appears to have been broadly distributed in the Indo-Pacific region but is believed to be found mostly near India. It has been listed as Critically Endangered by IUCN Red List of Threatened Species. Known as ‗Pala Sora‘ in the local parlance, the Pondicherry Shark is on the verge of extinction. It is among the 25 ‗most wanted lost‘ species that are the focus of Global Wildlife Conservation‘s ―Search for Lost Species‖ initiative. Hence, option c) is the correct answer.

Point (3) is correct. Ganges shark is endemic to India. It inhabits the River Hooghly in West Bengal, as well as the rivers Ganges, Brahmaputra, Mahanadi in the states of , Assam and Orissa. The Ganges shark is only found in rivers and possibly estuaries, with no confirmed records from oceans or seas. It is amongst the 20 most threatened shark species and is listed as a Critically Endangered species in the IUCN Red list. Recognising the dangers faced by this species, it is protected under Schedule I, Part II A of the Wildlife (Protection) Act of India, 1972.

Point (1) is incorrect-The dugong is a medium-sized marine mammal. It has been listed as Vulnerable by IUCN Red List of Threatened Species. Commonly known as "sea cows," dugongs graze peacefully on sea grasses in shallow coastal waters of the Indian and western Pacific Oceans.

Source: http://vajiramias.com/current-affairs/pondicherry- shark/5c1876a82099372ce5793044/

Q25. Consider the following rivers:

(1) Subansiri

(2) Dibang

(3) Lohit

(4) Barak

Which of the above are the tributaries of Brahmaputra? a) 1 and 3 only b) 1, 2 and 4 only c) 1, 2 and 3 only d) 1, 2, 3 and 4

Answer: c

Explanation:

The Tsangpo – Yarlung Zangbo, as it is known in China – flows into as the Siang, which meets two other rivers in Assam downstream to become the Brahmaputra. It turns towards south and enters Assam where it is joined by two mountain streams, the Lohit and the Dibang and thereafter it is known as Brahmaputra.

The principal tributaries of the river joining from right are the Lohit, the Dibang, the Subansiri, the Jiabharali, the Dhansiri, the Manas, the Torsa, the Sankosh and the Teesta whereas the Burhidihing, the Desang, the Dikhow, the Dhansiri and the Kopili joins it from left. Barak River is not a tributary of Brahmaputra. Recently, the water level of Siang river in Arunachal Pradesh has subsided after a scare triggered by an alert from China to India about the swelling of Tsangpo following heavy rain in Tibet. Hence, option (c) is the correct answer.

Source: http://vajiramias.com/current-affairs/siang/5c17d8392099372ce5792fc5/

Q26. Consider the following statements with reference to the ―Pradhan Mantri Annadata Aay Sanrakshan Abhiyan (PM-AASHA)‖:

(1) It is a new umbrella scheme aimed at ensuring remunerative prices to the farmers for their produce by providing Minimum Support Price (MSP) assurance.

(2) It is comprised of only two three sub-schemes that are Price Support Scheme (PSS), Price Deficiency Payment Scheme (PDPS) and pilot of Private Procurement & Stockist Scheme (PDPS).

Select the correct answer using the code given below: a) 1 only

b) 2 only c) Both 1 and 2 d) Neither 1 nor 2 Answer: c

Explanation:

Union Cabinet has approved a New Umbrella Scheme ―Pradhan Mantri Annadata Aay SanraksHan Abhiyan‖ (PM-AASHA). The scheme is launched by Union Government in September 2018.

Statement (1) is correct- Pradhan Mantri Annadata Aay Sanrakshan Abhiyan (PM- AASHA) will provide remunerative prices to the farmers for their produce by providing Minimum Support Price (MSP) assurance.

Statement (2) is correct- The new Umbrella Scheme is comprised of (1) Price Support Scheme (PSS), (2) Price Deficiency Payment Scheme (PDPS) and (3) Pilot of Private Procurement & Stockist Scheme (PPPS).

This is an unprecedented step taken by Govt. of India to protect the farmers‘ income which is expected to go a long way towards the welfare of farmers. Government has already increased the MSP of kharif crops by following the principle of 1.5 times the cost of production.

Source: http://vajiramias.com/current-affairs/pm-aasha/5c1880992099372ce5793063/

Q27. Which one of the following organizations issues the ―Human Development Index report‖ periodically? a) United Nations Development Programme (UNDP) b) United Nations Educational, Scientific and Cultural Organization (UNESCO) c) United Nations High Commissioner for Refugees (UNHCR ) d) United Nations Environment Programme (UNEP)

Answer: a

Explanation:

In the latest Human Development Index (HDI) released by the United Nations Development Programme (UNDP), India is ranked 130 out of 189 countries with the findings indicating a glaring inequality in the country though ―millions have been lifted

out of poverty‖. , Switzerland, Australia, Ireland and lead the ranking. Niger, the Central African Republic, South Sudan, Chad and Burundi have the lowest scores. South Asia experienced the fastest HDI growth among developing regions with a 45.3 % increase since 1990. South Asia has the widest gap between men and women in HDI at 16.3%.

Source: http://vajiramias.com/current-affairs/human-development-index- report/5c1886e62099372ce7423355/

Q28. With respect to ―Masala bonds‖, consider the following statements:

(1) Masala bonds are rupee-denominated borrowings instruments issued by Indian entities in overseas markets.

(2) These bonds are same like dollar bonds, where the borrower takes the currency risk.

(3) The first Masala bond was issued by the International Finance Corporation (IFC).

Which of the statements given above are correct? a) 1 and 2 only b) 2 and 3 only c) 1 and 3 only d) 1, 2 and 3

Answer: c

Explanation: In a bid to push Indian corporates to take the masala bond route, the Government of India has exempted all such bond issues until March 31, 2019, from withholding tax.

Statement (1) is correct- Masala bonds are essentially bonds issued by Indian companies, denominated in rupees, to overseas investors to attract funds for projects, especially in infrastructure. The objective of Masala Bonds is to fund infrastructure projects in India, fuel internal growth via borrowings and internationalise the Indian currency.

Statement (2) is incorrect- They are different from dollar bonds as in Dollar bonds; the borrower takes the currency risk. But in Masala bonds, the investors bear the currency risk.

Statement (3) is correct- The first Masala bond was issued by the International Finance Corporation (IFC), the private arm of the World Bank, in 2014 to fund

infrastructure projects in India. Masala, a Hindi word, means spices was used by IFC to evoke the cuisine and culture of India.

Indian corporates have for long borrowed in international credit markets — in the form of loans and bonds, and in various foreign currencies. This always came with the risk of the company having to pay more while repaying its debt. By issuing bonds in rupees in the overseas markets, the risk is transferred to investors who sign up for that, taking into account the growth prospects of the country and the issuing company. From the issuer‘s perspective, it means cheaper borrowings compared to raising funds in India besides diversifying its sources of fund-raising.

Source: http://vajiramias.com/current-affairs/masala- bond/5c1888332099372ce742335c/

Q29. Consider the following statements with reference to the Chiller Star Labelling Program:

(1) It is an initiative under the Ministry of New and Renewable energy.

(2) Chillers are used extensively for space conditioning of buildings and for industrial process cooling applications.

Which of the statements given above is/are correct? a) 1 only b) 2 only c) Both 1 and 2 d) Neither 1 nor 2 Answer: b

Explanation:

Statement (1) is incorrect- Union Ministry of Power, Government of India launched an ambitious program to encourage the deployment of Energy Efficient chiller systems in the country. The Chiller Star Labelling Program has been formulated by Bureau of Energy Efficiency (BEE). The program envisages providing star rating in terms of its energy performance. Initially, the program is launched on voluntary basis and will be valid upto 31st December 2020.

Statement (2) is correct- Chillers are used extensively for space conditioning of buildings and for industrial process cooling applications. Thus, it is important to optimize energy performance of chillers. Through this Program, it is estimated that more

than 500 million units of electricity would be saved in 2019 along with Green House Gases (GHG) reduction of 0.5 million-ton equivalent of CO2.

Bureau of Energy Efficiency has developed an online registration platform for easy and expeditious approval under this initiative. Based on the test certificate from designated agencies and after due verification from BEE, the star label (1 to 5) will be awarded, with 5 star being the most efficient chillers.

Source: http://vajiramias.com/current-affairs/chiller-star-labelling- program/5c1888732099372ce742335e/

Q30. Recently as a part of Capacity Building Programme the Indian Space Research Organisation (ISRO) has launched a space technology incubation centre at which of the following places? a) Agartala b) Shillong c) Kohima d) Dimapur

Answer: a

Explanation: The Indian Space Research Organisation (ISRO) has launched a space technology incubation centre in Tripura capital Agartala. The incubation centre will be located in the National Institute of Technology, Agartala. It is the first of six such centres planned nationally to build capacity in new locations (other being Jalandhar, Bhubaneswar, Tiruchi, Nagpur and Indore). The centres will bring out prototypes and innovations for ISRO in fields of electronics, propulsion and others.

Source: http://vajiramias.com/current-affairs/space-technology-incubation- centre/5c18acb82099372ce8f4dc5d/

Q31. With respect to the ―Institutes of Eminence (IOE)‖, consider the following statements:

(1) The IOEs will enjoy greater autonomy in terms of deciding their fee structure and course duration & course structure.

(2) They will be exempted from approvals of government or UGC for academic collaborations with foreign institutions.

(3) Only Public Institutes are eligible to apply under the Institutes of Eminence.

Which of the statements given above are correct? a) 1 and 2 only b) 2 and 3 only c) 1 and 3 only d) All of the above.

Answer: a

Explanation:

Indian Institute of Technology (IIT) Madras is planning to apply for the next list of Institutes of Eminence, after it couldn‘t be included in the first list released in July 2018.

Statement (1) is correct- The IOEs will enjoy greater autonomy in terms of deciding their fee structure and course duration & course structure.

Statement (2) is correct- They will be exempted from approvals of government or UGC for academic collaborations with foreign institutions.

Statement (3) is incorrect- In July 2018, the HRD Ministry granted ‗Institutions of Eminence‘ status to three public and three private institutes. Private institutes selected: Jio Institute by Reliance Foundation, Manipal Academy of Higher Education and BITS Pilani. Public institutes selected: IIT-Delhi, IIT-Bombay and the Bengaluru- based Indian Institute of Science (IISc).The move came as part of a scheme of the Human Resource Development (HRD) Ministry to select 20 IoEs — 10 public and 10 private — which will enjoy complete academic and administrative autonomy.

Source: http://vajiramias.com/current-affairs/institutions-of- eminence/5c19313f20993773066913f9/

Q32. The term ‗Novichok‘ is sometimes seen in the news in the context of___ a) Series of nerve agents b) Multi drug resistant bug c) Fissionable nuclear material d) A type of fermion

Answer: a

Explanation:

Novichok is a series of nerve agents developed by the Soviet Union and Russia between 1971 and 1993. They disrupt normal messaging from the nerves to the muscles. This causes muscles to become paralysed and can even cause death. These are among the most lethal chemical weapons and their use is banned under the Chemical Weapons Convention (CWC). Recently, Britain has charged two Russian intelligence officers for attempted murder of a former Russian spy and his daughter in England with a deadly nerve agent called Novichok. Hence, option a) is the correct answer.

Source: http://vajiramias.com/current-affairs/novichok/5c1863e62099372ce8f4dbb4/

Q33. What of the following best describes the term ―Standard Project Storm (SPS)‖, which was recently seen in news? a) Heaviest rainstorm that has occurred in a region as per rainfall records analysed through a hydro-meteorological approach. b) A widespread, long-lived, straight-line wind storm that is associated with a land- based, fast-moving group of severe thunderstorms. c) Data assimilation strategy with the goal to double current cyclone warning lead time through improved warning accuracy and reduced false alarms. d) None of the above.

Answer: a

Explanation:

The Central Water Commission has referred to extreme rainfall that occurred over two intense spells last month in Kerala as a ―standard project storm‖. Standard Project Storm (SPS) means the heaviest rainstorm that has occurred in a region as per rainfall records analysed through a hydro-meteorological approach. Technically, it is a storm which has occurred over an area, with the maximum depth in that area. For determining it, experts look at historical rainfall records, for all storms in the region and determine the depth and maximum area that it affected in that period. The ‗Probable Maximum Precipitation‘ for different basins provides the historical rainfall records. Hence, option a) is the correct answer.

Source: http://vajiramias.com/current-affairs/standard-project-storm- sps/5c187db32099372ce8f4dc09/

Q34. With reference to the ‗Total Expense Ratio (TER)‘, which of the following statements is/are correct?

(1) It is a measure of the total costs associated with managing and operating an investment fund.

(2) In India, the maximum TER that a fund can charge its investors is prescribed by Reserve Bank of India.

Select the correct answer using the code given below: a) 1 only b) 2 only c) Both 1 and 2 d) Neither 1 nor 2 Answer: a

Explanation:

Statement (1) is correct- Total Expense Ratio is a measure of the total costs associated with managing and operating an investment fund such as mutual funds. These costs consist primarily of management fees and additional expenses, such as trading fees, legal fees, auditor fees and other operational expenses.

Statement (2) is incorrect- In India, the maximum TER that a fund can charge its investors is prescribed by Securities and Exchange Board of India (SEBI). Recently, the SEBI lowered the total expense ratio (TER) for open-ended equity schemes, thereby making it less expensive for investors to invest in mutual funds. SEBI has capped the Total expense ratio at 1.05% for open-ended equity schemes with assets under management (AUM) in excess of ₹50,000 crore.

Source: http://vajiramias.com/current-affairs/total-expense-ratio- ter/5c18aa1f2099372ce8f4dc56/

Q35. ‗The Education Ministers Declaration 2018, Mendoza‘ recently in the news, is related to the affairs of which of the following? a) BRICS b) G7 c) G20 d) OECD

Answer: c

Explanation:

The Minister of State for Human Resource Development (HRD) attended the G-20 Education Ministers‘ Meeting held at Mendoza, Argentina. It was the first ever exclusive meeting of Education Ministers in the history of G-20. At the end of the meeting, G20 Education Ministers‘ Declaration 2018 was released. Its key highlights were:

(i) In line with the United Nations 2030 Agenda, it aims at ensuring inclusive and equitable quality education and promoting lifelong learning opportunities for all.

(ii) 21st century skills (related to societal and technological innovations such as Artificial Intelligence, Big Data and Internet of Things) should be considered in curriculum development.

(iii) Promote High participation of women in Science, Technology, Engineering, and Mathematics (STEM).

(iv) Create conditions for the emergence of innovative instruments for financing education.

(v) Facilitate the internationalisation of education, including the cross-border recognition of qualifications and skills, and the mobility of students, teachers etc.

Source: http://vajiramias.com/current-affairs/g20-education-ministers-declaration- 2018/5c186a3b2099372ce5793024/

Q36. Which of the following best describes ―Cyclone-30‖, which is recently seen in news? a) It is a strong tropical cyclone that devastated parts of Indonesia and China. b) It is used to produce radioisotopes for diagnostic and therapeutic use for cancer care. c) It is a software construct that bridges the gap between physical systems and the digital world. d) It is a handheld tool for multispectral analysis of tissue morphology.

Answer: b

Explanation:

Cyclotrons are used to produce radioisotopes for diagnostic and therapeutic use for cancer care. Recently, Cyclone-30, the biggest Medical Cyclotron Facility in India, became operational at Variable Energy Cyclotron Centre (VECC), Kolkata. It

is unit of Department of Atomic Energy (DAE). The facility will start regular production by the middle of the next year after the commissioning of the supporting nuclear systems and regulatory clearances. It will provide affordable radio isotopes and related radiopharmaceuticals for the entire country especially. It also has export potential for Germanium-68/Gallium-68 generator for in-situ production of Gallium-68 and Palladium- 103 isotopes, used for breast cancer diagnosis and prostate cancer treatment, respectively. Hence, option b) is the correct answer.

Source: http://vajiramias.com/current-affairs/cyclone-30/5c18aa802099372ce8f4dc58/

Q37. Which of the following pairs is/are correctly matched?

Diseases Agents

(1) Leptospirosis Bacteria

(2) Middle East Respiratory Syndrome Virus

(3) Malignant Catarrhal Fever Protozoa

Select the correct answer using the code given below: a) 1 only b) 1 and 2 only c) 2 and 3 only d) 1, 2 and 3

Answer: b

Explanation:

Pair (1) is correctly matched- Leptospirosis is an infectious disease caused by bacteria belonging to the genus Leptospira. It can be transmitted to humans through cuts and abrasions of the skin, or through the mucous membranes of the eyes, nose and mouth with water contaminated with the urine of infected animals. Although, all wild and domestic mammals can harbour the bacteria that cause leptospirosis, but Rodents are the primary source of infection to human beings. Human-to-human transmission occurs only very rarely. Leptospirosis occurs worldwide, but is most

prevalent in tropical and subtropical regions. Outbreaks of leptospirosis are common when flooding occurs, such as following a typhoon or very heavy seasonal rains, because of exposure to contaminated water.

Pair (2) is correctly matched- Middle East respiratory syndrome (MERS) is a viral respiratory disease. Coronaviruses are a large family of viruses that can cause diseases ranging from the common cold to Severe Acute Respiratory Syndrome (SARS). It is caused by a novel coronavirus (Middle East Respiratory Syndrome Coronavirus, or MERS‐CoV) that was first identified in Saudi Arabia in 2012.

MERS-CoV is a zoonotic virus, which means it is a virus that is transmitted between animals and people. Dromedary camels are a major reservoir host for MERS-CoV and an animal source of infection in humans. The virus does not pass easily from person to person unless there is close contact, such as providing unprotected care to an infected patient. No vaccine or specific treatment is currently available.

Pair (3) is not correctly matched- Malignant Catarrhal Fever (MCF), an infectious viral disease originally from South Africa that affects cattle. It is caused by a group of ruminant gamma herpes viruses including Alcelaphine gamma herpesvirus 1 (AlHV- 1) and Ovine gamma herpesvirus 2 (OvHV-2). The virus is transmitted from pregnant or recently-lambed sheep or goats to cattle although several months may elapse between such contact and clinical disease. It has neither a cure nor a vaccination that can prevent its spread. Recently, Malignant Catarrhal Fever (MCF), a rare and deadly livestock disease has resurfaced in Tumakuru district, Karnataka.

Source: http://vajiramias.com/current-affairs/leptospirosis/5c1854572099372ce5792fea/ http://vajiramias.com/current-affairs/middle-east-respiratory- syndrome/5c1873852099372ce579303d/ http://vajiramias.com/current-affairs/malignant-catarrhal-fever- mcf/5c1879ad2099372ce8f4dbfb/

Q38. Diviseema Island, which was in the news recently, is located in which of the following states / Union territories? a) West Bengal b) Karnataka c) Andaman and Nicobar Islands d) Andhra Pradesh

Answer: d

Explanation:

Diviseema Island is located in the Krishna District of Andhra Pradesh. It is located in the delta area formed at Puligadda (Avanigadda), where the is divided into two before merging into the Bay of Bengal. The island comprises three mandals: Koduru, Avanigadda and Nagayalanka. It is a rare ecological region with dense mangrove cover. The decline of frogs began when local communities hunted down frogs and shipped them illegally to China, depriving the snakes. In India, a ban on frog exports came into force in 1987. This disrupted the food chain due to the virtual collapse of their prey base on the island due to which snakes started entering agriculture fields and human habitations. Hence, option (d) is the correct answer.

Source: http://vajiramias.com/current- affairs/diviseema/5c18e473209937730669139a/

Q39. Consider the following fauna of India:

(1) Indian Roofed Turtle

(2) Black Spotted Turtle

(3) Black Softshell Turtle

Which of the above is/are declared extinct in the wild by the IUCN Red list of Threatened species? a) 1 and 3 only b) 2 and 3 only c) 3 only d) None of the above

Answer: c

Explanation:

Point (1) is incorrect- As the name suggest, Indian Roofed Turtle have ‗roof‘ like shape on the top of their shell which is their distinct feature. It is listed as Least Concern by the IUCN Red list of Threatened species. It is found in the major rivers of South Asian countries (Ganges, Brahmaputra and drainages) in the Pakistan, India, Bangladesh and Nepal. Listed on Convention on International Trade in Endangered Species (CITES) Appendix I.

Point (2) is incorrect- According to a recent report by TRAFFIC, an international network monitoring trade in wildlife, India accounts for 29% of black spotted turtles seized from across seven countries in South Asia. It is native to South Asia (Bangladesh; India; Nepal; Pakistan). In India, it is distributed across the north, northeast and a few parts of central India. It is listed as vulnerable by the IUCN Red list of Threatened species. It is a medium-sized freshwater turtle having a black shell with yellow streaks. Listed on Convention on International Trade in Endangered Species (CITES) Appendix I.

Point (3) is correct- The black softshell turtle or Bostami turtle is a species of freshwater turtle found in India and Bangladesh. The black softshell turtle (Nilssonia nigricans) figures in the International Union for Conservation of Nature‘s (IUCN) Red List as ―extinct in the wild‖. But a few temple ponds in Assam and Bangladesh are bringing these turtles back from the brink. India hosts 28 species of turtles, of which 20 are found in Assam. But consumption of turtle meat and eggs, silt mining, encroachment of wetlands and change in flooding pattern have had a disastrous impact on the State‘s turtle population.

Source: http://vajiramias.com/current-affairs/indian-roofed- turtles/5c18557b2099372ce74232ca/ http://vajiramias.com/current-affairs/black-spotted-turtles/5c1931fd2099377308fff9b5/

Q40. Which of the following pairs is/are correctly matched?

Tribes States (1) Chola Naikar Andhra Pradesh (2) Hatti

(3) Tiwa Assam

Select the correct answer using the code given below: a) 1 only b) 1 and 2 only c) 2 and 3 only d) 1, 2 and 3

Answer: c

Explanation:

Pair (1) is not correctly matched- Chola Naikar is a tribe found mainly in the Silent Valley National Park of Kerala. They speak Canarese, a dialect of Kannada. They are said to have migrated from Mysore forests. They are one of the last remaining hunter-gatherer tribes of the region. They are called Chola Naikar because they inhabit the interior forests. ‗Chola‘ or ‗shoals‘ means deep ever green forest, and ‗naikar‘ means King. They also call themselves as ‗Malanaikar‘ or ‗Sholanaikar‘. In the aftermath of floods, Kerala‘s reclusive Chola Naikar tribe is leaving the forests for the plains.

Pair (2) is correctly matched- Himachal Pradesh government has urged Union Home Ministry to accord tribal status to the Hatti community of Sirmour district. The community has been accorded this status in Uttarakhand. The Hattis are a unique people living in the trans-Giri area of Himachal Pradesh. Some of them are in Uttarakhand, enjoying the Scheduled Tribe status and the resultant benefits. Age- old traditions such as animal sacrifice, folk dances and their unique attire-Thalka or Lohiya not only make them a distinct community but also one which is yet to embrace modernization.

Pair (3) is correctly matched- Tiwa (Lalung) is indigenous community inhabiting the states of Assam and and also found in some parts of Arunachal Pradesh and Manipur in Northeast India. They are recognized as a Scheduled tribe within the State of Assam. The Wanchuwa festival is one of the most important celebrations in the life of the Tiwa tribe of Assam.

Source: http://vajiramias.com/current-affairs/chola-naikar- tribe/5c184dcc2099372ce74232a9/ http://vajiramias.com/current-affairs/schedule-tribe-status/5c18733c2099372ce579303b/

Q41. With reference to ―Atal Bimit Vyakti Kalyan Yojna‖, consider the following statements:

(1) It is a scheme for Insured Persons (IP) covered under the Employees‘ State Insurance Act, 1948.

(2) The scheme is a relief payable in cash directly to their Bank Account in case of unemployment and while they search for new engagement.

Which of the statements given above is /are correct? a) 1 only b) 2 only

c) Both 1 and 2 d) Neither 1 nor 2

Answer: c

Explanation:

The Employee State Insurance Corporation (ESIC) has approved a Scheme named ―Atal Bimit Vyakti Kalyan Yojna‖.

Statement (1) is correct- Employees' State Insurance Corporation under Ministry of Labour and Employment has recently rolled out Atal Bimit Vyakti Kalyan Yojna. It is a scheme for Insured Persons (IP) covered under the Employees‘ State Insurance Act, 1948.

Statement (2) is correct- This scheme is a relief payable in cash directly to their Bank Account in case of unemployment and while they search for new engagement. The scheme has been rolled out considering the change in employment pattern in India from a long-term employment to fixed short term engagement in the form of contract.

Source: http://vajiramias.com/current-affairs/atal-bimit-vyakti-kalyan- yojna/5c18b4fe2099372ce8f4dc65/

Q42. With reference to ―Loo Review campaign‖, consider the following statements:

(1) It is a joint initiative of Ministry of Drinking Water and Sanitation and Ministry of Information and Broadcasting.

(2) It aims to encourage local guides in India to rate and review public toilets on Google Maps.

Which of the statements given above is/are correct? a) 1 only b) 2 only c) Both 1 and 2 d) Neither 1 nor 2 Answer: b

Explanation:

Statement (1) is incorrect- The Ministry of Housing and Urban Affairs, under the aegis of Swachh Bharat Mission –Urban (SBM – U), has partnered with Google to launch the Loo Review campaign. Objective of the joint campaign is to increase the awareness and ease of locating public toilets across India.

Statement (2) is correct- The campaign aims to encourage Local Guides in India to rate and review public toilets on Google Maps, and use the hashtag #LooReview for the chance to be featured on Google Local Guides‘ social channels.

The feedback provided by local guides through the Loo Review campaign will press upon the Urban Local Bodies to take proactive steps to improve public toilet facilities across the country.‖ 500+ cities in India with more than 30,000 toilets with the name of ―SBM Toilet‖ are currently live on Google Maps.

Source: http://vajiramias.com/current-affairs/looreview- campaign/5c19285220993773071c42ac/

Q43. Exercise Yudh Abhyas 2018 was a joint military exercise between India and which one of the following countries? a) Russia b) Indonesia c) Sri Lanka d) United States of America

Answer: d

Explanation:

Exercise Yudh Abhyas 2018 is a joint military exercise of Indian and US armies, commenced at Chaubattia, Uttarakhand. This was the 14th edition of the joint exercise hosted alternately between the two countries. Hence, option d) is the correct answer.

Source: http://vajiramias.com/current-affairs/yudh-abhyas- 2018/5c189adc2099372ce8f4dc41/

Q44. With reference to ―National Digital Communications Policy -2018‖, consider the following statements:

(1) Universal broadband connectivity at 50 Mbps speed to every citizen by the year 2022 is one of the key objectives of the policy.

(2) The policy advocates the establishment of a National Digital Grid by creating a National Fibre Authority

(3) Under this policy the name of Telecom Commission will be changed to Digital Communications Commission.

Which of the statements given above is/are correct? a) 1 and 2 only b) 2 only c) 2 and 3 only d) 1, 2 and 3

Answer: d

Explanation:

Union Cabinet has approved the National Digital Communications Policy-2018 (NDCP- 2018), in place of the existing National Telecom Policy- 2012, to provide broadband access to every citizen at 50 Mbps speed by the year 2022.

Statement (1) is correct- The key objectives of the policy are Broadband for all; Creating four million additional jobs in the Digital Communications sector; Enhancing the contribution of the Digital Communications sector to 8% of India's GDP from ~ 6% in 2017; Propelling India to the Top 50 Nations in the ICT Development Index of ITU from 134 in 2017; Enhancing India's contribution to Global Value Chains; and Ensuring Digital Sovereignty.

Statement (2) is correct- The policy advocates:-Establishment of a National Digital Grid by creating a National Fibre Authority; Establishing Common Service Ducts and utility corridors in all new city and highway road projects; Creating a collaborative institutional mechanism between Centre, States and Local Bodies for Common Rights of Way, standardization of costs and timelines; Removal of barriers to approvals; and Facilitating development of Open Access Next Generation Networks.

Statement (3) is correct. To ensure effective implementation and monitoring of the National Digital Communications Policy-2018, the government has decided to re- designate Telecom Commission -- the apex decision making body at the telecom department -- as the Digital Communications Commission. The change in name has been brought in line with objectives of NDCP (National Digital Communications Policy) to project telecommunications as a business enabling segment that permeates across various sectors.

Source: http://vajiramias.com/current-affairs/national-digital-communications-policy- 2018/5c190ee420993773066913da/

Q45. Recently NITI Aayog has released a report titled, ―Mission on shifting cultivation: towards a transformational approach‖ which calls for recognising land for shifting cultivation as ―Agricultural Land‖ under agro-forestry. This practice of shifting cultivation is known by different names in different regions of India.

In this regard consider the following pairs:

Shifting Cultivation States (1) Jhum Nagaland (2) Podu Madhya Pradesh (3) Bewar Kerala

Which of the above pairs is/are correctly matched? a) 1 only b) 1 and 3 only c) 2 and 3 only d) 1, 2 and 3

Answer: a

Explanation:

Shifting cultivation has been characterised by rotation of fields rather than rotation of crops, the exclusive use of human labour, use of dibble stick or hoe, and short periods of occupancy alternating with long fallow periods to assist the regeneration of vegetation.

Shifting cultivation, locally referred to as jhum cultivation, is considered as an important mainstay of food production for a considerable population in northeast India in States like Arunachal Pradesh, Nagaland, Mizoram, Meghalaya, Tripura and Manipur. In South India, shifting farming is called as ―Kumari‖, ‗Podu‘ in Odisha and Andhra Pradesh, watra in south east Rajasthan. In Madhya Pradesh, the farming is named as ―Bewar‖. Hence, option a) is the correct answer.

Source: http://vajiramias.com/current-affairs/jhum- cultivation/5c18a10f2099372ce579308d/

Q46. Recently NASA has launched ICESat-2 mission. It is related to which of the following? a) Measuring changes in Earth's ice sheets. b) Collecting samples from Ryugu. c) Exploring Pluto and Kuiper Belt d) The study of intermittent liquid water on the Martian Surface

Answer: a

Explanation:

National Aeronautics and Space Administration (NASA) launches satellite ICESat- 2 to measure Earth's sea ice, glaciers and oceans. ICESat-2 is NASA‘s most advanced space laser satellite with the objective of tracking ice loss around the world and improve forecasts of sea level rise as the climate warms. It will help estimate elevation change in the Greenland and Antarctic sheets. The preceding mission, ICESat, launched in 2003 and ended in 2009. It revealed that sea ice was thinning, and ice cover was disappearing from coastal areas in Greenland and Antarctica. ICESat-2 will orbit Earth carrying a photon-counting laser altimeter, ATLAS (Advanced Topographic Laser Altimeter System).

Hayabusa 2 is a Japan Aerospace Exploration Agency‘s (JAXA) mission planning to rendezvous with an asteroid, land a small probe and three miniature rovers on the surface, and then return samples to Earth. Hayabusa 2 is projected to arrive at asteroid 1999 JU3, also called ―Ryugu,‖ in 2018. (Hayabusa was earlier mission, Hayabusa 2 is recent mission)

The New Horizons spacecraft launched in 2006 was the first mission to Pluto, completing the space-age reconnaissance of the planets that started 50 years earlier. It was also the first mission to explore the solar system's recently-discovered "third zone," the region beyond the giant planets called the Kuiper Belt.

Source: http://vajiramias.com/current-affairs/icesat-2/5c18951a2099372ce742336d/

Q47. Consider the following statements regarding Mobilise Your City (MYC) initiative:

(1) It is a part of an international initiative which is supported by the French and the German Governments.

(2) It aims at supporting all metropolitan cities in India to reduce their Green House Gas (GHG) emissions related to urban transport.

Which of the statements given above is/are correct? a) 1 only b) 2 only c) Both 1 and 2 d) Neither 1 nor 2 Answer: a

Explanation:

Statement (1) is correct- Mobilise Your City (MYC) is a part of an international initiative which is supported by the French and the German Governments and was launched at 21st Conference of Parties (COP21) meeting in December, 2015.

Statement (2) is incorrect- The MYC aims at supporting three pilot cities viz. Nagpur, Kochi and Ahmedabad in their efforts to reduce their Green House Gas (GHG) emissions related to urban transport by implementing urban mobility plans at local level and to help India at national level to improve their sustainable transport policy.

Recently India and France have signed an implementation agreement on ―Mobilise Your City‖ (MYC).

Source: http://vajiramias.com/current-affairs/mobilise-your-city- myc/5c186a8c2099372ce5793027/

Q48. Consider the following statements about the Local Treatment of Urban Sewage Streams for Healthy Reuse (LOTUS HR):

(1) It is an initiative of the Ministry of Environment, Forest and Climate Change.

(2) It aims to recover nutrients and energy from the urban waste water, thus converting drain into profitable mines.

Which of the statements given above is/are correct? a) 1 only b) 2 only

c) Both 1 and 2 d) Neither 1 nor 2 Answer: b

Explanation:

Statement (1) is incorrect- The project Local Treatment of Urban Sewage Streams for Healthy Reuse (LOTUS HR) is an Indo- joint project funded by Department of Biotechnology, Government of India and the Government of Netherlands.

Statement (2) is correct- The project aims to demonstrate a novel holistic (waste-) water management approach, that will produce clean water that can be reused for various proposes (e.g. industry, agriculture, construction etc.), while simultaneously recovering nutrients and energy from the urban waste water, thus converting drain into profitable mines. Special attention will be paid to pathogen removal and removing conventional and emerging pollutants.

Recently Union Science & Technology minister inaugurates two unique projects including Local Treatment of Urban Sewage Streams for Healthy Reuse project, in Delhi.

Source: http://vajiramias.com/current-affairs/lotus-hr/5c18ac2a2099372ce8f4dc5b/

Q49. Wind Augmentation Purifying Unit (WAYU) which was in the news recently has been developed by which of the following organizations? a) Aerial Delivery Research & Development Establishment (ADRDE) b) Centre for Air Borne System (CABS) c) Centre for Environment and Energy Development (CEED) d) Council of Scientific and Industrial Research – National Environmental Engineering Research Institute (CSIR-NEERI)

Answer: d

Explanation:

Union Science & Technology Minister recently inaugurated air pollution control device – Wind Augmentation Purifying Unit (WAYU) – for traffic junctions at ITO intersection and Mukarba Chowk in Delhi. WAYU has been indigenously developed by Council of Scientific and Industrial Research – National Environmental Engineering

Research Institute (CSIR-NEERI) as a part of Technology Development Project being funded by Department of Science and Technology.

The device works on two principles mainly (i) Wind generation for dilution of air pollutants and (ii) Active Pollutants removal (The device has filters for Particulate Matter removal). The prototype device has the capacity to purify air in an area of 500-meter square. Government is also in the process of developing a bigger version of WAYU which can purify air in an area of 10,000-meter square.

Source: http://vajiramias.com/current-affairs/wind-augmentation-purifying-unit- wayu/5c19049920993773066913ce/

Q50. Coral reefs are one of the most biologically diverse marine eco-systems on the Earth. In which of the following places coral reefs can be found in India?

(1)

(2) Malvan

(3) Lakshadweep Islands

(4) Gulf of Mannar

Select the correct answer using the codes given below: a) 1 and 3 only b) 2 and 4 only c) 1, 3 and 4 only d) 1, 2, 3 and 4

Answer: d

Explanation:

A team from National Centre for Coastal Research, Chennai is working on coral restoration in the Gulf of Mannar region. Earlier, they successfully transplanted and nurtured corals in the Lakshadweep region. Coral reefs are present in the areas of Gulf of Kutch, Gulf of Mannar, Andaman & Nicobar, Lakshadweep Islands and Malvan.

Corals are animals which live in a symbiotic relationship with microscopic algae called zooxanthellae (which live within the coral tissue). The zooxanthellae convert sunlight into food, providing corals with up to 90 per cent of their energy needs. They also give

corals much of their colour. Bleaching occurs when stressful conditions, such as heat, cause this relationship to break down, resulting in the corals expelling their zooxanthellae. This leaves the coral tissue mostly transparent, revealing the coral's bright white skeleton. This loss of their symbiotic algae means bleached corals are essentially starving. Currently, there are many global initiatives such as Global Coral Reef Monitoring Network (GCRMN), Global Coral Reef Alliance (GCRA), and International Coral Reef Initiative (ICRI) playing an important role in monitoring the reef zones and raising awareness in the public. Hence, option d) is the correct answer.

Source: http://vajiramias.com/current-affairs/coral-reefs/5c1895c22099372ce579307f/

Q51. Paris climate change agreement adopted on December 12, 2015, during the United Nations Framework Convention on Climate Change‘s (UNFCCC) 21st Conference of Parties (COP21), is a historic turning point for global climate action.

Which of the following initiatives is/ are associated with or launched to support the Paris agreement?

(1) Green Climate Fund (GCF)

(2) Global Solar Council (GSC)

(3) Global Climate Action Summit

(4) Under2 Coalition

Select the correct answer using the code given below: a) 3 only b) 2 and 3 only c) 2, 3 and 4 only d) 1, 2, 3 and 4

Answer: d

Explanation:

Point (1) is correct- The Green Climate Fund (GCF) is a global fund within the UNFCCC framework to support the developing countries limit or reduce their greenhouse gas (GHG) emissions and adapt to climate change. It was set up at COP 16 held in Cancun in 2010, by the 194 countries who are parties to the United Nations Framework Convention on Climate Change (UNFCCC), as part of the Convention‘s financial mechanism. GCF launched its initial resource mobilization in 2014. When the Paris Agreement was reached in 2015, the GCF was given an important role in supporting the goal of keeping climate change well below 2 degrees Celsius. Recently, Officials overseeing the Green Climate Fund (GCF) have approved more than $1 billion in new investments in a meeting held in Bahrain.

Point (2) is correct- The Global Solar Council is an international non-profit association of the national, regional and international associations in solar energy and the world‗s leading corporations. It was launched at the COP 21 climate change meeting in Paris. The GSC came into being as International Coalition of more than 30 nations, utilising maximum solar energy, decided to harness the renewable energy for the greater good.

Point (3) is correct- The Global Climate Action Summit (GCAS) 2018, in , was the first ever global climate summit, focused exclusively on the actions and ambitions of businesses, cities, states and citizens worldwide. It bought together US and international leaders who are committed to turning the tide in

the fight against climate change. A successful GCAS 2018 will now help to reassure the international community that this will not decelerate the ―decarbonisation‖ movement. Participation of non-state actors made the Global Climate Action Summit different. This Summit and its Call to Action make an important contribution towards achieving the collective goal: to keep global temperatures to 1.5 degrees Celsius in line with the Paris Agreement.

Point (4) is correct- The Under2 Coalition is driven by a group of ambitious state and regional governments committed to keeping global temperature rises to well below 2°C. The coalition is made up of more than 220 governments who represent over 1.3 billion people and 43% of the global economy. The Climate Group is the Secretariat to the Under2 Coalition and works with governments to accelerate climate action through three work streams. It supports governments to develop robust medium and long-term (2050) emissions reduction plans in line with the goals of the Paris Climate Agreement. Hence, option d) is the correct answer.

Source: http://vajiramias.com/current-affairs/green-climate- fund/5c1b340e20993706fc3da061/

Q52. With respect to ―National Company Law Tribunal (NCLT)‖, consider the following statements:

(1) Under Companies Act the term of office of chairperson and members of all NCLTs is three years or 65 years, whichever is earlier.

(2) The decisions of the National Company Law Appellate Tribunal may be appealed to the Supreme Court of India.

Which of the statements given above is/are correct? a) 1 only b) 2 only c) Both 1 and 2 d) Neither 1 nor 2

Answer: b

Explanation:

National Company Law Tribunal (NCLT) is a quasi-judicial body that adjudicates issues relating to Indian companies. It was constituted by the Central Government under section 408 of the Companies Act, 2013 w.e.f. 1st June 2016. In the first phase

the Ministry of Corporate Affairs set up eleven Benches including one Principal Bench at New Delhi. These Benches are headed by the President and 16 Judicial Members and 09 Technical Members at different locations.

Statement (1) is incorrect- Section 413 of the Companies Act fixes the term of office of chairperson and members of all NCLTs at five years or 65 years, whichever is earlier.

Statement (2) is correct- Decisions of the NCLT may be appealed to the National Company Law Appellate Tribunal (NCLAT). The decisions of NCLAT may be appealed to the Supreme Court of India.

Source: http://vajiramias.com/current-affairs/national-company-law-tribunal- nclt/5c19aad72099377308fff9bf/

Q53. Which of the following rivers of India flowing towards the west, drain into the Arabian Sea?

(1) Pampa

(2) Palar

(3) Vaigai

Select the correct answer using the code given below: a) 1 only b) 2 and 3 only c) 1 and 3 only d) 1, 2 and 3

Answer: a

Explanation:

Recently Parts of terracotta artefacts of archaeological value have been unearthed from the banks of the Pampa River at Aranmula, giving further support to the theory that ‗Pampa Valley Civilisation‘ flourished.

Point (1) is correct- Pampa River originates from Annamalai hills and flowing through Kerala joins Vembanad Lake. The Pamba, 176 km in length is the third longest river in Kerala. The Pamba River rises in the Peermedu Plateau.

Point (2) is incorrect- Palar River rises beyond Talagvare village in the Kolar district of Karnataka and drains into the Bay of Bengal near Kuvattur in Tamil Nadu. On its way, the river Palar receives two important tributaries namely, the Poini on the left bank and the Cheyyar on the right bank. The Water Resources Department (WRD) of Tamil Nadu is expected to begin work on check dams across Palar to help recharge groundwater.

Point (3) is incorrect- Vaigai River originates from the hills of Varshanad in Madurai district (Tamil Nadu) and drains into the . The Suruliyar and the Manjalar, the two important left bank tributaries. The Vaigai basin is an important basin among the 12 basins lying between the Cauvery and Kanyakumari. This basin is bounded by the Varushanadu hills, the Andipatti hills, the Cardaman hills and the Palani hills on the West and by the and Palk Bay on the East. Hence, option a) is the correct answer.

Source: http://vajiramias.com/current-affairs/pampa- river/5c19aa952099377308fff9bd/

Q54. What is ―SATAT‖, which was in news recently? a) It is an app to explore local geology virtually on smartphone for free. b) It is a high resolution earth observation satellite. c) It is an indigenously developed low-cost hearing screening device for newborns. d) It is an initiative to promote Compressed Bio-Gas.

Answer: d

Explanation:

Recently Union Petroleum Minister launches SATAT initiative to promote Compressed Bio-Gas as an alternative, green transport fuel. SATAT is an acronym for

―Sustainable Alternative Towards Affordable Transportation‖. This initiative has been launched with PSU Oil Marketing Companies (OMCs) inviting Expression of Interest (EoI) from potential entrepreneurs to set up Compressed Bio-Gas (CBG) production plants and make available CBG in the market for use in automotive fuels. The Working Group on Biofuels, set up under the National Policy on Biofuels 2018, is in the process of finalising a pan-India pricing model for Compressed Bio-Gas. Hence, option d) is the correct answer.

Bio-gas is produced naturally through a process of anaerobic decomposition from waste / bio-mass sources like agriculture residue, cattle dung, municipal solid waste etc. After purification, it is compressed and called Compressed Bio-Gas (CBG), which has pure methane content of over 95%. CBG is exactly similar to the commercially available natural gas in its composition and energy potential.

Source: http://vajiramias.com/current-affairs/satat/5c19b01c20993773071c42d4/

Q55. With reference to ―Global RE-INVEST‖, consider the following statements:

(1) It was organized by the Union Ministry of New and Renewable Energy.

(2) It aims to scale up renewable energy and connect the global investment community with Indian energy stakeholders.

Which of the statements given above is/are correct? a) 1 only b) 2 only c) Both 1 and 2 d) Neither 1 nor 2

Answer: c

Explanation:

Recently Prime Minister inaugurated the 2nd Global RE-Invest (Renewable Energy Investors‘ Meet and Expo) in New Delhi.

Statement (1) is correct- It was organized by the Union Ministry of New and Renewable Energy from 2nd to 5th October 2018 in New Delhi.

Statement (2) is correct- It aims at building upon the success of RE-INVEST 2015 to accelerate the worldwide effort to scale up renewable energy and connect the global investment community with Indian energy stakeholders.

Source: http://vajiramias.com/current-affairs/re-invest/5c19b6852099377308fff9de/

Q56. The ‗Delhi Declaration on Renewable Energy‘ recently in the news, is related to the affairs of: a) Association of Southeast Asian Nations (ASEAN) b) South Asian Association for Regional Cooperation (SAARC) c) Indian Ocean Rim Association (IORA) d) World Trade Organization (WTO)

Answer: c

Explanation:

Recently, Indian Ocean Rim Association (IORA) adopted the Delhi Declaration on Renewable Energy in the Indian Ocean Region, post the 2nd IORA Renewable Energy Ministerial Meeting held at the 2nd Global Re-Invest India-ISA Partnership Renewable Energy Investor‘s Meet & Expo in Greater Noida. It calls for collaboration among IORA member states in meeting the growing demand for renewable energy in the Indian Ocean littorals, development of a common renewable energy agenda for the Indian Ocean region and promote regional capacity building. The declaration also calls for promotion of technology development and transfer, strengthening of public private partnerships in renewable energy and collaboration among IORA member states and the member nations of the International Solar Alliance (ISA). Hence, option c) is the correct answer.

Indian Ocean Rim Association (IORA) is an inter-governmental organisation which was established in 1997. IORA was set up with the objective of strengthening regional cooperation and sustainable development within the Indian Ocean Region with 21 Member States and 7 Dialogue Partners. Members are India, Australia, Iran IR, Indonesia Thailand, Malaysia, South Africa, Mozambique, Kenya, Sri Lanka, Tanzania, Bangladesh, Singapore, Mauritius, Madagascar, UAE, Yemen, Seychelles, Somalia, Comoros and Oman. IORA‘s apex body is the Council of Foreign Ministers (COM) which meets annually. IORA secretariat is based in Mauritius and is overseen by a secretary-general who is appointed for a three-year period.

Source: http://vajiramias.com/current-affairs/delhi-declaration-on-renewable- energy/5c19c103209937730669143b/

Q57. With reference to ―UNESCO Global Geoparks‖, consider the following statements:

(1) Lake in Maharashtra and St. Mary‘s Island in coastal Karnataka are the only UNESCO Global Geoparks in India.

(2) A World Heritage Site or Biosphere Reserve cannot apply for the UNESCO Global Geopark status.

Which of the statements given above is/are not correct? a) 1 only b) 2 only c) Both 1 and 2 d) Neither 1 nor 2 Answer: c

Explanation:

Statement (1) is incorrect- in Maharashtra and St. Mary‘s Island and Malpe beach in coastal Karnataka are the Geological Survey of India (GSI)

candidates for UNESCO Global Geopark Network status. They are single, unified geographical areas where sites and landscapes of international geological significance are managed with a holistic concept of protection, education and sustainable development. UNESCO Global Geopark status does not imply restrictions on any economic activity inside the site where that activity complies with indigenous, local, regional and/or national legislation. As of November 2015, there are 119 UNESCO Global Geoparks spread across 33 countries, mostly in Europe and China. No Indian site is in the list till now. A UNESCO Global Geopark is given this designation for a period of four years after which the functioning of the site re-examined during a revalidation process.

Statement (2) is incorrect- A World Heritage Site or Biosphere Reserve can apply for the UNESCO Global Geopark status. However, a clear evidence has to be provided on how UNESCO Global Geopark status will add value by being both independently branded and in synergy with the other designations.

Source: http://vajiramias.com/current-affairs/global-geopark- network/5c19b4b22099377306691418/

Q58. ―Amrutanubhava‖ a philosophical work written by which of the following Bhakti Saints? a) Dnyaneshwar b) Chaitanya c) Vallabhacharya d) Ramanuja

Answer: a

Explanation:

Recently Vice-President of India inaugurated the ‗World Peace Monument‘ dome – World‘s largest dome – on the 150 th birth anniversary of Mahatma Gandhi. It is located at the Maharashtra Institute of Technology (MIT)‘s World Peace University (MIT-WPU) campus at Loni Kalbhor near Pune. The dome is built atop the MIT World Peace Library and the World Peace Prayer Hall, which are named after the 13th century poet-saint and philosopher ‗Dnyaneshwar‘ a pivotal figure of the Bhakti movement in Maharashtra. He is also known as Jnaneshwar or Mauli (1275–1296) was a 13th-century Marathi saint, poet, philosopher and yogi of the Nath tradition whose Dnyaneshwari (a commentary on the Bhagavad Gita) and Amrutanubhav are

considered to be milestones in Marathi literature. Dnyaneshwar composed Dnyaneshwari in the year 1290, a commentary on Bhagavad Gita which later became a fundamental text of the Varkari sect. Hence, option a) is the correct answer.

Source: http://vajiramias.com/current-affairs/world-peace- monument/5c19b4f72099377308fff9d7/

Q59. Which one of the following is the best description of the term ‗Optical Tweezers‘ that was in news recently? a) An instrument that will look for new fundamental particles at the Large Hadron Collider (LHC). b) A method of using light to capture and manipulate tiny objects. c) A method to monitor ultrafine dust particles. d) None of the above

Answer: b

Explanation:

Recently, Arthur Ashkin of U.S. has been awarded the Prize for his invention of ―optical tweezers‖ that grab particles, atoms, viruses and other living cells with their laser beam fingers. Optical tweezers are scientific instruments that use a highly focused laser beam to provide an attractive or repulsive force (depending on the relative refractive index between particle and surrounding medium) to physically hold and move microscopic objects similar to tweezers. This method of using light to capture and manipulate tiny objects has changed the way of studying microscopic life.

Optical tweezers have been particularly successful in studying a variety of biological systems in recent years. A major breakthrough came in 1987 when Mr. Ashkin used the tweezers to capture living bacteria without harming them. Mr. Ashkin is the oldest winner of a Nobel prize, beating out American Leonid Hurwicz who was 90 when he won the 2007 Economics Prize.

Source: http://vajiramias.com/current-affairs/nobel-prize-in- physics/5c19b6d62099377306691422/

Q60. Consider the following statements about Chirped Pulse amplification (CPA):

(1) It is a technique for amplifying pulses to very high optical intensities while avoiding excessive nonlinear pulse distortions or optical damage.

(2) It is widely used in corrective eye surgery.

Which of the statements given above is/are correct? a) 1 only b) 2 only c) Both 1 and 2 d) Neither 1 nor 2

Answer: c

Explanation:

Statement (1) is correct- Gerard Mourou (of France) and Donna Strickland (of ) have been awarded for helping develop a method to generate ―ultra-short optical pulses‖, the shortest and most intense laser pulses ever created. A technique for amplifying pulses to very high optical intensities while avoiding excessive nonlinear pulse distortions or optical damage

Statement (2) is correct- Laser light can be emitted in short pulses, but they can‘t be amplified beyond a point without destroying the material. Mourou and Strickland‘s technique known as Chirped Pulse Amplification (CPA), helped solved this problem. It is now widely used in corrective eye surgery.

Donna Strickland (of Canada) is third woman to win the Nobel Prize in Physics, after Maria Goeppert-Mayer and Marie Curie.

Source: http://vajiramias.com/current-affairs/nobel-prize-in- physics/5c19b6d62099377306691422/

Q61. Recently the Government of India and the Asian Development Bank (ADB) have signed an agreement to establish India‘s first Global Skills Park in which of the following States? a) Kerala b) Madhya Pradesh c) Gujarat d) Andhra Pradesh

Answer: b

Explanation:

Government of India and the Asian Development Bank (ADB) has signed a $ 150 Million Loan Agreement to establish India‘s first Global Skills Park in State of Madhya Pradesh. The First Multi-Skills Park in India will enhance the quality of Technical and Vocational Education and Training (TVET) System in the State and create a more skilled workforce. The new GSP campus will be established in Bhopal, Madhya Pradesh. The Project will consist of core Advanced Training Institutes including the Center for Occupational Skills Acquisition and the Center for Advanced Agricultural Training as well as other support services focusing on entrepreneurship, training of trainers etc. Hence, option b) is the correct answer.

Source: http://vajiramias.com/current-affairs/global-skills-park- gsp/5c19b93720993773071c42e8/

Q62. Which one of the following is the motive of ‗Udyam Abhilasha‘ that was in news recently? a) An environmental awareness campaign at the national level. b) A skill development campaign exclusively for women. c) A caregiver training scheme to care for Person with Disabilities (PwD) and their families. d) A national level entrepreneurship awareness campaign.

Answer: d

Explanation: SIDBI has launched a National Level Entrepreneurship Awareness Campaign, Udyam Abhilasha in 115 Aspirational Districts identified by NITI Aayog in 28 States and reaching to around 15,000 youth. Udyam Abhilasha is a National Level Entrepreneurship Awareness Campaign. Objective of the campaign is to inspire rural youth in aspirational districts to be entrepreneurs by assisting them to set up their own enterprise. SIDBI has partnered with CSC e-Governance Services India Limited, a Special Purpose Vehicle, (CSC SPV) set up by the Ministry of Electronics & IT, Govt. of India for implementing the campaign through their CSCs. Further, CSC Village Level entrepreneurs (VLEs) would play role of catalyst for these aspiring entrepreneurs. Hence, option d) is the correct answer.

Source: http://vajiramias.com/current-affairs/udyam- abhilasha/5c19b9942099377306691429/

Q63. Digi Yatra, often mentioned in the news is an initiative of which of the following Ministries? a) Ministry of Road Transport and Highways b) Ministry of Civil Aviation c) Ministry of Railways d) Ministry of Shipping

Answer: b

Explanation:

Recently Union Civil aviation Ministry Released the policy on Biometric based Digital processing of passengers at airports called ―Digi Yatra‖. ―Digi Yatra‖ initiative, or biometric-enabled digital processing of passengers, would enable travellers to enter the airport building by scanning a QR code on their mobile phones, after undergoing facial recognition.

Once inside the airport, a passenger would be able to self-check-in, drop baggage, pass through e-gates to access security and embarkation areas with just a facial scan, thus removing the need to produce a boarding pass at every step. The facility, which is voluntary, would require passengers to initially register themselves at a web portal by providing an identity proof. The travellers would then be required to undergo a one-time verification at an airport following which, the individual‘s facial identity would be captured and mapped onto a newly created ―Digi Yatra‖ profile and a distinct identification number would be generated. Passengers would then have to provide this identification number at the time of purchasing an air ticket, to avail the paperless access facility. Hyderabad and Bengaluru airports would be the first to implement the digital processing of passengers. The ―Digi Yatra‖ programme would comply with the European Union‘s Data Protection Regulation. Hence, option b) is the correct answer.

Source: http://vajiramias.com/current-affairs/digi- yatra/5c19c178209937730669143e/

Q64. Recently, India's first "Methanol Cooking Fuel Program" has been launched in which of the following states?

a) Punjab b) Maharashtra c) Assam d) Tamil Nadu

Answer: c

Explanation:

Assam Petro-chemicals a state-owned company, launched Asia's first cannisters based and India's first "Methanol Cooking Fuel Program". The safe handling cannister based cooking stoves are from Swedish Technology and through a Technology transfer a large-scale cooking stove manufacturing plant will come up in India in the next 18 months producing 10 lakh Cookstoves and 1 Crore Cannisters per year. 500 households inside the Assam Petro Complex will be the first pilot project, scaling it to 40,000 households in Uttar Pradesh, Maharashtra, Gujarat, Telangana, Goa and Karnataka. Its objective is to reduce import of crude and to provide clean, import substitute, cost effective and pollution free cooking medium. Hence, option c) is the correct answer.

Source: http://vajiramias.com/current-affairs/methanol-cooking-fuel-program-of- india/5c19c71e20993773071c42f7/

Q65. Consider the following statements about GenomeAsia 100k:

(1) Its objective is to sequence the whole genomes of 100k Asians.

(2) India is not part of the ‗GenomeAsia 100k‘ project.

Which of the statements given above is/are correct? a) 1 only b) 2 only c) Both 1 and 2 d) Neither 1 nor 2 Answer: a

Explanation:

Statement (2) is incorrect- The Prime Minister‘s Science, Technology and Innovation Advisory Council (STIAC) in its first meeting announced that India will be a part of the ‗GenomeAsia 100k‘ project.

Statement (1) is correct- Objective of the GenomeAsia 100k project, first announced in 2016, is to sequence the whole genomes of 100k Asians, including 50,000 Indians. It is being led out of the National Technological University (NTU), Singapore.

The project is similar to the projects in the , China, Japan and Australia and the information from this will be used to improve health by designing ‗personalised medicine. The Ministry of Health and Family Welfare and the Department of Biotechnology would be closely associated with the project.

Source: http://vajiramias.com/current- affairs/genomeasia100k/5c19fcc82099377308fffa57/

Q66. Consider the following statements with reference to the Strategic Petroleum Reserves:

(1) International Energy Agency (IEA) members maintain emergency oil reserves equivalent to at least 90 days of net imports.

(2) Indian Strategic Petroleum Reserves Ltd (ISPRL) has constructed three strategic petroleum reserves at Visakhapatnam, Mangaluru and Padur.

Which of the statements given above is/are correct? a) 1 only b) 2 only c) Both 1 and 2 d) Neither 1 nor 2 Answer: c

Explanation:

Strategic petroleum reserves are essentially huge stockpiles of crude oil; an emergency store of crude oil to tide over severe supply shocks of this critical fuel.

Statement (1) is correct- International Energy Agency (IEA) members maintain emergency oil reserves equivalent to at least 90 days of net imports.

Statement (2) is correct- Indian Strategic Petroleum Reserves Ltd (ISPRL) is a special purpose vehicle under the Oil and Gas Ministry for constructing strategic petroleum reserves in India. ISPRL has constructed three strategic petroleum reserves at Visakhapatnam on the East Coast, and at Mangaluru and Padur on the West Coast. These facilities, with total capacity of 5.33 million tonnes, can meet 10 days of India‘s crude oil requirements. In July 2018, the government approved the construction of two more reserves at Chandikhol in Odisha and Padur in Karnataka, having an aggregate capacity of 6.5 million tonnes. The new facilities can provide additional supply for about 12 days. India is the world‘s third largest energy consumer after the US and China. But India is dependent on imports for more than 80 % of her energy needs.

Source: http://vajiramias.com/current-affairs/indian-strategic-petroleum- reserves/5c1af48420993706fc3da007/

Q67. What is ‗Cambrian Explosion‘, which was seen in the news recently? a) Sudden appearance in the fossil record of complex animals with mineralised skeletal remains of 541 million years ago. b) The most dangerous type of volcanic eruption. c) The USSR detonated the largest nuclear weapon ever tested and created the biggest man-made explosion in history. d) Severe meteor explosion ever recorded due to Chelyabinsk meteor

Answer: a

Explanation:

Researchers have found the oldest clue yet to the mystery of animal life in ancient rocks and oils, including those from India, dating back at least 100 million years before the famous Cambrian explosion of animal fossils. Researchers at the University of California, Riverside in the U.S. tracked molecular signs of animal life, called biomarkers, as far back as 660-635 million years ago during the Neoproterozoic era.

In ancient rocks and oils from India, Oman, , they found a steroid compound produced only by sponges, which are among the earliest forms of animal life. The ―Cambrian Explosion‖ refers to the sudden appearance in the fossil record of complex animals with mineralised skeletal remains of 541 million years ago. The

biomarker they identified, a steroid compound named 26-methylstigmastane (26-mes), has a unique structure that is currently only known to be synthesised by certain species of modern sponges called demosponges. This steroid biomarker is the first evidence that demosponges, and hence multicellular animals, were thriving in ancient seas at least as far back as 635 million years ago. Hence, option a) is the correct answer.

Source: http://vajiramias.com/current-affairs/26- methylstigmastane/5c1af5a920993706ff7274ef/

Q68. What is ‗Feihong-98 (FH-98)‘, which was recently in the news? a) World‘s largest unmanned transport aircraft b) Electric plane tested by NASA. c) Space Observatory launched by China. d) Medium lift launch vehicle designed and manufactured by SpaceX.

Answer: a

Explanation:

China has successfully tested the world‘s largest unmanned transport drone which can carry a payload of 1.5 tonnes. A large commercial drone Feihong-98 (FH-98) developed and modified by the China Academy of Aerospace Electronics Technology made a test flight in north China‘s Inner Mongolia autonomous region.

Feihong-98 (FH-98) is now the world‘s largest unmanned transport aircraft, with a maximum payload of up to 1.5 tonnes. It was adapted from the prototype of the Shifei Y5B, a China-developed transport plane. The FH-98 features simple take-off and landing, simple operation, advanced technology, at an affordable cost. Hence, option a) is the correct answer.

Source: http://vajiramias.com/current-affairs/feihong-98-fh- 98/5c1afd7420993706ff7274fc/

Q69. Consider the following statements about Bedaquiline:

(1) It is a bactericidal drug which belongs to a new class of antibiotics.

(2) It is recommended by WHO specifically for the treatment of patients suffering from multidrug-resistant tuberculosis (MDR-TB) and latent TB infection.

Which of the statements given above is/are correct? a) 1 only b) 2 only c) Both 1 and 2 d) Neither 1 nor 2 Answer: a

Explanation:

In a recent Rapid Communication, the World Health Organisation (WHO) has included bedaquiline drug, specifically developed for treating multidrug-resistant TB (resistant toisoniazid and rifampicin) patients, in the fully oral regimen.

Statement (1) is correct- Bedaquiline is a bactericidal drug which belongs to a new class of antibiotics (diarylquinolines).

Statement (2) is incorrect- It is recommended by World Health Organisation (WHO) specifically for the treatment of patients suffering from multidrug-resistant tuberculosis (MDR-TB). It is not recommended for treating latent TB infection. The drug is made available only to the Centre, which in turn rations it to the States. It is not available for private purchase because of concerns of drug-resistance by Mycobacterium tuberculosis, the bacterium that causes TB.

Source: http://vajiramias.com/current- affairs/bedaquiline/5c1a007a2099377308fffa5f/

Q70. Recently, in which of the following places in India, a research team has found the burial places of megalithic culture? a) Bhimbetka b) Lakhudiyar c) Panduvaraguddum d) Barabar

Answer: c

Explanation:

A review of a megalithic site done by researchers at Morlabanda in Anantapur district of Andhra Pradesh has noticed 18 dolmenoid cists (burial places) surrounded by slab circles on a big granite hill also locally called ‗Panduvaraguddum‘. A megalith is a large stone that has been used to construct a structure or monument, either alone or together with other stones. The word megalithic describes structures made of such large stones without the use of mortar or concrete, representing periods of prehistory characterised by such constructions. The construction of these structures took place mainly in the Neolithic period and continued into the Chalcolithic period and the Bronze Age. Hence, option c) is the correct answer.

Source: http://vajiramias.com/current-affairs/megalithic- site/5c19d90720993773071c4311/

Q71. Consider the following statements:

(1) They are slightly smaller than their African counterparts.

(2) Listed in Schedule I of Wildlife (Protection) Act 1972 and as Endangered on IUCN Red List.

(3) At present Gir National Park and Wildlife Sanctuary is the only abode for them.

(4) Canine Distemper Virus (CDV) found to be the cause of recent deaths of many.

Above statements describes which of the following animals? a) Indian Wild Ass b) Cheetah c) Blackbuck d) Asiatic Lion

Answer: d

Explanation:

According to Gujarat state government, Canine Distemper Virus (CDV) was found in some of the 23 Asiatic lions which died in Gujarat's Gir forest since September 12. Canine distemper is a contagious viral disease that attacks the respiratory, gastrointestinal and nervous systems of a wide variety of animal species, including dogs, coyotes, foxes, pandas and wolves.

Asiatic lions are slightly smaller than African lions. The most striking morphological character, which is always seen in Asiatic lions, and rarely in African lions, is a

longitudinal fold of skin running along its belly. According to the Asiatic lion census conducted in august 2017, the lion population was estimated at 650 wild individuals. Listed in Schedule I of Wildlife (Protection) Act 1972, in Appendix I of CITES and as Endangered on IUCN Red List. Asiatic lions were once distributed upto the state of West Bengal in east and Rewa in Madhya Pradesh, in central India. At present Gir National Park and Wildlife Sanctuary is the only abode of the Asiatic lion. The last surviving population of the Asiatic lions is a compact tract of dry deciduous forest and open grassy scrublands in southwestern part of Saurashtra region of Gujarat. Hence, option d) is the correct answer.

Source: http://vajiramias.com/current-affairs/asiatic- lions/5c19b0fa2099377308fff9d2/ http://vajiramias.com/current-affairs/canine-distemper/5c19bffd2099377308fff9ef/

Q72. Which of the following is/are brackish water lake(s) in India?

(1)

(2)

(3)

Select the correct answer using the code given below: a) 1 only b) 2 and 3 only c) 1 and 2 only d) 1, 2 and 3

Answer: c

Explanation:

Brackish water is water that has more salt than freshwater, but not as much as seawater.

Point (1) is correct: Chilika Lake is a brackish water lagoon, spread over the Puri, Khurda and Ganjam districts of Odisha. It is located at the mouth of the Daya River, flowing into the Bay of Bengal. It is the largest coastal lagoon in India and the second largest lagoon in the world after the New Caledonian barrier reef in New

Caledonia. In 1981, Chilika Lake was designated the first Indian wetland of international importance under the Ramsar Convention.

Point (2) is correct: Pulicat Lagoon is the second largest brackish water lagoon in India, after Chilika Lake. Located on the border of Andhra Pradesh and Tamil Nadu, Pulicat Lake is India's second largest lagoon. Although 90% of the lake falls in Andhra Pradesh, the other 10% comes under Protected Areas of Tamil Nadu. Pulicat Lake is fed by River Kalangi and River Arani. The lagoon is separated from the Bay of Bengal by a barrier island which is the Sriharikota.

Point (3) is incorrect: Mansar Lake is one of the largest freshwater in the Shivalik range of the Jammu region; the lake is facing an existential threat due to human intervention and climate change.

Other notable brackish bodies of water in India are Kutch Salt Marsh and . Hence, option c) is the correct answer.

Source: http://vajiramias.com/current-affairs/chilika- lake/5c1b64dc2099370b77a895a3/

Q73. 'DHARMA GUARDIAN-2018' was the first ever joint military exercise involving India and which of the following countries? a) China b) Indonesia c) Australia d) Japan

Answer: d

Explanation:

India and Japan will hold the first ever joint military exercise 'DHARMA GUARDIAN-2018' involving the Indian Army and Japan Ground Self Defence Force at Counter Insurgency Warfare School, Vairengte, India. Hence, option d) is the correct answer.

Source: http://vajiramias.com/current-affairs/dharma-guardian- 2018/5c1aff3920993706fc3da01c/

Q74. Consider the following statements about petroglyphs:

(1) They are patterns and shapes made by many large rocks and boulders over the ground.

(2) They were recently discovered in the Saurashtra region of Gujarat.

Which of the statements given above is/are correct? a) 1 only b) 2 only c) Both 1 and 2 d) Neither 1 nor 2 Answer: d

Explanation:

Statement (2) is incorrect- Recently, petroglyphs were discovered in the Ratnagiri and Rajapur areas in the Konkan region of Maharashtra. Those rock carvings which might date back to 10,000 BC, depict animals like hippopotamuses and rhinoceroses which aren‘t found in that region of India.

Statement (1) is incorrect- Petroglyphs are images created by removing part of a rock surface by incising, picking, carving, or abrading, as a form of rock art. Scholars often use terms such as "carving", "engraving" to refer such images. Petroglyphs are found worldwide, and are often associated with prehistoric peoples.

The term petroglyph should not be confused with Petrograph and Petroforms. Petrograph is an image drawn or painted on a rock face. Petroforms are patterns and shapes made by many large rocks and boulders over the ground.

Source: http://vajiramias.com/current- affairs/petroglyphs/5c1b0fb920993706ff72750c/

Q75. With reference to Azad Hind Government, consider the following statements:

(1) It was founded on 21st October, 1943 in occupied Singapore.

(2) The government of Azad Hind had its own currency, court and civil code.

Which of the statements given above is/are correct? a) 1 only b) 2 only

c) Both 1 and 2 d) Neither 1 nor 2 Answer: c

Explanation:

The Prime Minister of India hoisted the National flag and unveiling the plaque to celebrate the 75th anniversary of the formation of Azad Hind Government on 21st October, 2018, at the Red Fort, Delhi.

Statement (1) is correct- The Azad Hind Government was founded on 21st October, 1943 in occupied Singapore. It was established by Indian nationalists-in- exile during the Second World War in Singapore and was inspired by Netaji Subhash Chandra Bose who was the leader of Azad Hind Government and also the Head of State of this Provisional Indian Government-in-exile. It was a part of the freedom movement, originating in 1940s outside India with a purpose of allying with Axis powers to free India from British rule.

Statement (2) is correct- The government of Azad Hind had its own currency, court and civil code, however, it lacked large and definite areas of sovereign territory until Japan gave it nominal authority of the Andaman and Nicobar Islands in 1943 and the occupation of parts of Manipur and Nagaland.

Source: http://vajiramias.com/current-affairs/azad-hind- government/5c1b102e20993706fe821107/

Q76. Which of the following pairs is/are correctly matched?

Spacecraft Purpose

1. BepiColombo Comprehensive study of Mercury 2. Voyager 2 Exploring the outer solar system

3. HAYABUSA 2 Probe Ryugu asteroid‘s surface

Select the correct answer using the code given below: a) 1 only b) 1 and 2 only c) 2 and 3 only

d) 1, 2 and 3

Answer: d

Explanation:

Point (1) is correct: European and Japanese space agencies announced that an Ariane 5 rocket successfully lifted a BepiColombo spacecraft carrying two probes into orbit for a joint mission to Mercury. Mercury, which is only slightly larger than Earth‘s moon, has a massive iron core about which little is known. Researchers are also hoping to learn more about the formation of the solar system from the data gathered by the BepiColombo mission. BepiColombo spacecraft has begun its seven-year journey to Mercury before arriving at its destination in December 2025.

Point (2) is correct: NASA‘s Voyager 2 probe is approaching interstellar space and has detected an increase in cosmic rays that originate outside our solar system. Launched in 1977, Voyager 2 is a little less than 17.7 billion kilometres from Earth, or more than 118 times the distance from Earth to the Sun. Since 2007 the probe has been travelling through the outermost layer of the heliosphere – the vast bubble around the Sun and the planets dominated by solar material and magnetic fields. Voyager scientists have been watching for the spacecraft to reach the outer boundary of the heliosphere, known as the heliopause. Once Voyager 2 exits the heliosphere, it will become the second human-made object, after Voyager 1, to enter interstellar space.

Point (3) is correct: The MASCOT lander, part of the Hayabusa2 probe successfully landed on Ryugu asteroid‘s surface. The Hayabusa2 spacecraft is a mission to analyse the composition of asteroid 1999 JU3 (Ryugu) and return samples from it to Earth using the MASCOT (Mobile Asteroid Surface Scout) lander. The mission will shed light on the origins of the solar system. Hayabusa2 was launched by JAXA, the Japanese space agency. German space agency DLR has developed the MASCOT lander and CNES supplied the MicrOmega instrument, antennas and electrical power system.

Source: http://vajiramias.com/current-affairs/nasas-voyager- 2/5c19d40820993773071c430d/ http://vajiramias.com/current-affairs/bepicolombo/5c1b217a20993706fc3da050/ http://vajiramias.com/current- affairs/hayabusa2mascot/5c19bd592099377308fff9e7/

Q77. ‗India Remembers‘ project is a nation-wide initiative, designed to raise awareness of the contribution and sacrifices made by India‘s servicemen and women since: a) First World War b) Second World War c) Royal Indian Navy mutiny d) Revolt of 1857

Answer: a

Explanation:

‗India Remembers‘ was a nation-wide initiative designed to raise awareness of the contribution and sacrifices made by India‘s servicemen and women since 1914.The India Remembers project is a joint endeavor of the USI and the Commonwealth War Graves Commission (CWGC) and a part of the ‗India and the Great War‘ Centenary Commemoration project initiated by the USI in 2014 with the support of the Ministry of External Affairs and in close association with the British High Commission. In line with this, the ‗India Remembers‘ project initiated by the USI proposed that the marigold flower join the poppy as a uniquely Indian symbol of remembrance. The ubiquitous marigold was the symbol of India‘s sacrifices when the world marked the centenary of the end of the First World War on November 11. Hence, option a) is the correct answer.

Source: http://vajiramias.com/current-affairs/india-remembers- project/5c1b2ea920993706fc3da058/

Q78. With reference to the ‗Neelkuruinji flowers‘, sometimes seen in the news, which of the following statements is/are correct?

(1) It is a shrub prominently found only in the Shevroys in the Eastern Ghats.

(2) The blue flowers of Neelakurinji blossoms only once in 6 years.

Select the correct answer using the code given below: a) 1 only b) 2 only c) Both 1 and 2

d) Neither 1 nor 2 Answer: d

Explanation:

Statement (1) is incorrect- Neelakurinji flowers are a shrub prominently found in the shola forests of the Western Ghats. Besides it is also seen in the Shevroys in the Eastern Ghats.

Statement (2) is incorrect- The blue flowers of Neelakurinji blossoms only once in 12 years. Core area of Kurinjimala Sanctuary, in Idukki district of Kerala, protects the kurinji. Nilgiri Hills (Literally meaning the blue mountains), got their name from the blue flowers of Neelakurinji. The Paliyan tribes (in Tamil Nadu) use it as a reference to calculate their age. Plants that bloom at long intervals like Neelakurinji are known as plietesials. Masting means a mass seeding phenomenon i.e. "Synchronous production of seed at long intervals by a population of plants". Neelakurinji is an example of it.

Source: http://vajiramias.com/current- affairs/neelakurinji/5c1b3cb72099370b76395983/

Q79. Black Sea is a border of which of the following countries?

(1) Romania

(2) Turkey

(3) France

(4) Georgia.

Select the correct answer using the code given below: a) 1 and 2 only b) 1 and 3 only c) 2 and 4 only d) 1, 2 and 4

Answer: d

Explanation:

Researchers have found an ancient Greek trading ship dating back more than 2,400 years virtually intact at the bottom of the Black Sea, making it the oldest known shipwreck.

The Black Sea a marginal sea of the Atlantic Ocean is located between Eastern Europe and Western Asia. Neighboring countries: It is bordered by six countries – Romania and Bulgaria to the west; Ukraine, Russia, and Georgia to the north and east; and Turkey to the south. The Black Sea drains into the Mediterranean Sea, via the Aegean Sea and various straits. The Bosphorus Strait connects it to the Sea of Marmara, and the Strait of the Dardanelles connects that sea to the Aegean Sea region of the Mediterranean. The Black Sea is also connected to the Sea of Azov by the Strait of Kerch. Five major rivers empty into the Black Sea, the largest of which is the Danube River. The Black Sea is the world's largest body of water with a meromictic basin. A meromictic lake has layers of water that do not intermix. Hence, option d) is the correct answer.

Source: http://vajiramias.com/current-affairs/black- sea/5c1b419b2099370b763959aa/

Q80. The Living Planet Report, released every two years is published by which of the following:

a) International Union for Conservation of Nature (IUCN) b) World Wide Fund (WWF) c) United Nations Development Programme (UNDP) d) United Nations Environment Programme (UNEP)

Answer: b

Explanation:

According to the Living Planet report 2018 released by the World Wide Fund for Nature (WWF), India‘s soil biodiversity is in grave danger. The key drivers of biodiversity decline remain overexploitation and agriculture. Indeed, of all the plant, amphibian, reptile, bird and mammal species that have gone extinct since AD 1500, 75% were harmed by overexploitation or agricultural activity or both.

The Living Planet Report, WWF‘s flagship publication released every two years. The Living Planet Report 2018 is the twelfth edition of the report. It is a comprehensive study of trends in global biodiversity and the health of the planet. The Living Planet Index (LPI), provided by the Zoological Society of London (ZSL), shows an overall decline of 60% in the population sizes of vertebrates between 1970 and 2014 in other words, an average drop of well over half in less than 50 years. Hence, option b) is the correct answer.

Source: http://vajiramias.com/current-affairs/living-planet-report- 2018/5c1b854120993711121c1842/

Q81. Consider the following statements about Early Warning Dissemination System (EWDS), Odisha:

(1) It is part of the last-mile connectivity programme under National Cyclone Risk Mitigation Project.

(2) It is a collaborative effort of the Central and State governments and has been implemented under the assistance of World Bank.

Which of the statements given above is/are correct? a) 1 only b) 2 only c) Both 1 and 2

d) Neither 1 nor 2 Answer: c

Explanation:

Statement (1) is correct- The Odisha government launched the Early Warning Dissemination System (EWDS), the first-of-its-kind technology in India. It‘s a part of the last-mile connectivity programme under National Cyclone Risk Mitigation Project.

Statement (2) is correct- The EWDS, is a collaborative effort of the Central and State governments, has been implemented under the assistance of World Bank.

Its objective is to simultaneously warn coastal communities and fisherfolk about impending cyclone and tsunami through siren towers. Fishermen fishing in deep sea can also be reached via mass SMS on their mobile phones through EWDS. EWDS comprises technologies such as satellite-based mobile data voice terminals, digital mobile radio, mass messaging system and universal communication interface for interoperability. Sirens will go off from 122 towers installed along the 480-km-long coast of the State if a button is pressed in the State emergency centre in Bhubaneswar.

Source: http://vajiramias.com/current-affairs/early-warning-dissemination- system-ewds-odisha/5c1b803420993711121c183b/

Q82. Consider the following statements with reference to the Electoral Bond Scheme 2018:

(1) Only recognised national Political Parties shall be eligible to receive the Electoral Bonds.

(2) Electoral Bonds shall be valid for one year from the date of issue.

(3) State Bank of India (SBI) is the only authorised bank to issue such bonds.

Which of the statements given above are not correct? a) 1 and 2 only b) 2 and 3 only c) 1 and 3 only d) 1, 2 and 3

Answer: a

Explanation:

Recently, Government of India has notified the Electoral Bond Scheme 2018.

Statement (1) is incorrect- Only the Political Parties registered under Section 29A of the Representation of the People Act, 1951 and which secured not less than one per cent of the votes polled in the last General Election to the House of the People or the Legislative Assembly of the State shall be eligible to receive the Electoral Bonds.

Statement (2) is incorrect- The Electoral Bonds shall be encashed by an eligible Political Party only through a Bank account with the Authorized Bank. Electoral Bonds shall be valid for fifteen calendar days from the date of issue and no payment shall be made to any payee Political Party if the Electoral Bond is deposited after expiry of the validity period.

Statement (3) is correct- State Bank of India (SBI) is the only authorised bank to issue such bonds. The Electoral Bond deposited by an eligible Political Party in its account shall be credited on the same day. The bonds will be issued in multiples of ₹1,000, ₹10,000, ₹1 lakh, ₹10 lakh and ₹1 crore and will be available at specified branches of State Bank of India. They can be bought by the donor with a KYC- compliant account.

Source: http://vajiramias.com/current-affairs/electoral-bond-scheme- 2018/5c1b6e2a2099370b79fd1487/

Q83. Which of the following pairs is/are correctly matched?

Nuclear Power Plants States 1. Kaiga Tamil Nadu 2. Rawatbhatta Karnataka 3. Kudankulam Kerala

Select the correct answer using the code given below: a) 1 only b) 1 and 2 only c) 2 and 3 only d) None of the above

Answer: d

Explanation:

Point (1) is incorrect: Kaiga Atomic Power Station (KAPS) has set a new world record among pressurised heavy water reactors (PHWRs) with one of its units operating uninterrupted for 895 days as of 26th October. The Kaiga Atomic Power Station is a nuclear power generating station situated at Kaiga, near the river Kali, in Uttar Kannada district of Karnataka, India. The plant has been in operation since 2000 and is operated by the Nuclear Power Corporation of India. Kaiga now stands first in the world for continuous operation with regard to PHWRs and second among all nuclear power reactors. The Advanced Gas Cooled Reactor (AGR), Heysham II-8 of the United Kingdom, holds the overall record of 940 days of continuous operation.

Point (2) is incorrect: The Rajasthan Atomic Power Station is located at Rawatbhata in the state of Rajasthan, India. Bharat Heavy Electricals (BHEL) is currently installing 2 nuclear units each of 700 Mw capacity each at Kakrapara in Gujarat and Rawatbhata in Rajasthan.

Point (3) is incorrect: Kudankulam Nuclear Power Plant is the single largest nuclear power station in India, situated in Koodankulam in the Tirunelveli district of the southern Indian state of Tamil Nadu. The Supreme Court recently directed the Nuclear Power Corporation of India (NPCIL) to set up a facility for safe storage of radio-active spent nuclear fuel at Kudankulam Nuclear Power Plant (KKNPP) by April 2022. Hence, option d) is the correct answer.

Source: http://vajiramias.com/current-affairs/kaiga-atomic-power- station/5c1b5b162099370b77a8956c/

Q84. Consider the following statements about ―Scheme for Promotion of Academic and Research Collaboration (SPARC)‖:

(1) Its objective is to improve the research ecosystem of India‘s higher educational institutions by facilitating academic and research collaborations between Indian Institutions and the best institutions in the world.

(2) Indian Institute of Technology (IIT) Kharagpur is the National Coordinating Institute to implement the SPARC programme.

Which of the statements given above is/are correct? a) 1 only b) 2 only c) Both 1 and 2 d) Neither 1 nor 2 Answer: c

Explanation:

Recently Union HRD Minister launched the web portal of the ―Scheme for Promotion of Academic and Research Collaboration (SPARC)‖ in New Delhi.

Statement (1) is correct- SPARC objective is to improve the research ecosystem of India‘s higher educational institutions by facilitating academic and research collaborations between Indian Institutions and the best institutions in the world.

Statement (2) is correct- Indian Institute of Technology (IIT) Kharagpur is the National Coordinating Institute to implement the SPARC programme. Only such Indian institutes can apply which are in top 100 NIRF ranking or top 100 NIRF subject ranking. For foreign universities the benchmark is either top 500 of QS ranking or top 200 of QS subject ranking.

Source: http://vajiramias.com/current-affairs/scheme-for-promotion-of-academic- and-research-collaboration-sparc/5c1b56982099370b79fd13f5/

Q85. Consider the following statements with reference to the Tagore Award for Cultural Harmony:

(1) The annual award was instituted by the Government of India during the commemoration of 150th Birth Anniversary of Gurudev Rabindranath Tagore.

(2) The award is given to individuals and organizations in recognition of their outstanding contribution to cultural harmony.

(3) The award is open to only Indians.

Which of the statements given above is/are correct? a) 1 and 2 only

b) 2 and 3 only c) 1 and 3 only d) 1, 2 and 3

Answer: a

Explanation:

Winner of the Tagore Award for Cultural Harmony for the years 2014, 2015 & 2016 has been announced. The Tagore Award for Cultural Harmony for the years 2014, 2015 & 2016 is being conferred on the doyen of Manipuri dance Sh. Rajkumar Singhajit Singh; Chhayanaut (a cultural organization of Bangladesh) and on one of India‘s greatest sculptors, Sh. Ram Vanji Sutar, respectively.

Statement (1) is correct- The annual award was instituted by the Government of India during the commemoration of 150 th Birth Anniversary of Gurudev Rabindranath Tagore.

Statement (2) is correct- The award is given to individuals and organizations in recognition of their outstanding contribution to cultural harmony.

Statement (3) is incorrect- The award is open to all persons regardless of nationality, race, language, caste, creed or sex.

The award jury was headed by Prime Minister Narendra Modi and included Chief Justice of India Justice Ranjan Gogoi, former chief election commissioner N Gopalaswami and national vice president of the BJP Vinay Sahasrabuddhe.

The award carries an amount of Rs. 1 crore, a citation in a scroll, a plaque as well as an exquisite traditional handicraft/ handloom item.

The first Tagore Award was conferred on Pt. Ravi Shankar, the Indian Sitar Maestro in 2012 and second was conferred on Zubin Mehta (Western classical conductor) in 2013.

Source: http://vajiramias.com/current-affairs/tagore-award-for-cultural- harmony/5c1b59352099370b79fd141d/

Q86. Which one of the following is the best description of the term ‗Syncells‘ that was in the news recently? a) Microscopic devices made of graphene and a tiny electronic device used to produce armies of tiny robots no larger than living cells.

b) Giant cube-shaped neutrino detector of the size of a three-storey building. c) World's fastest supercomputer. d) None of the above

Answer: a

Explanation:

Syncells are microscopic devices made of graphene and a tiny electronic device used to produce armies of tiny robots no larger than living cells. These robots could find uses in monitoring a range of data, from infections in the human body to conditions in oil and gas pipelines. MIT scientists have developed robots no bigger than a cell that could be used to monitor conditions inside an oil or gas pipeline, or to search out disease while floating through the bloodstream.

The key to making ―syncells‖ in large quantities lies in controlling the natural fracturing process of atomically-thin, brittle materials. The process, called ―autoperforation‖, directs the fracture lines so that they produce miniscule pockets of a predictable size and shape. Embedded inside these pockets are electronic circuits and materials that can collect data. The system, developed by researchers uses a two-dimensional form of carbon called graphene, which forms the outer structure of the tiny syncells.

The giant cube-shaped neutrino detector of the size of a three-storey building and the largest liquid-argon neutrino detector in the world, called ProtoDUNE, the first of the two prototypes for an even larger detector to be built for the Deep Underground Neutrino Experiment (DUNE), just recorded its first particle tracks.

Source: http://vajiramias.com/current- affairs/syncells/5c1b51a52099370b76395a06/

Q87. A Geographical Indication (GI) is an indication used on products that have a specific geographical origin and possess qualities or a reputation that are due to that origin. Which of the following recently got GI tag in India?

(1) Alphonso Mango from Ratnagiri

(2) Kadaknath chicken of Jhabua

(3) Warangal dhurries

Select the correct answer using the code given below: a) 1 only

b) 1 and 2 only c) 2 and 3 only d) 1, 2 and 3

Answer: d

Explanation:

Point (1) is correct: Alphonso Mango from Ratnagiri, Sindhudurg, Palghar, and Raigad districts of Maharashtra, is registered as Geographical Indication (GI). The king of mangoes, Alphonso, better known as ‗Hapus‘ in Maharashtra, is in demand in domestic and international markets for its taste, pleasant fragrance and vibrant colour. It is exported to various countries including Japan, Korea and Europe. New markets such as USA and Australia have recently opened up.

Point (2) is correct: Kadaknath chicken breed is unique for its black colour due to its black-feathers. Its black colour stems from the deposition of melanin pigment. It is native tribal districts of Jhabua, Alirajpur and parts of Dhar in Madhya Pradesh.

Point (3) is correct: Dhurries is popular traditional thick cotton rug in which weavers create beautiful patterns and dye them using vegetable colours, which are washed in flowing water after the printing process. Warangal Carpets is an established industry in the state of Telangana, with a large community of skilled laborers and artisans for both dyeing the yarn to weave. Marketing the carpets directly can lead to better profits for the struggling sector in Telangana. Hence, option d) is the correct answer.

Source: http://vajiramias.com/current-affairs/geographical-indication- gi/5c19c6a920993773071c42f5/

Q88. With reference to the Prithvi-II missile, consider the following statements:

(1) It is a surface-to-air missile with a strike range of 2000 km.

(2) It is developed by DRDO of India under the Integrated Guided Missile Development Program.

Which of the statements given above is/are correct? a) 1 only b) 2 only c) Both 1 and 2 d) Neither 1 nor 2

Answer: b

Explanation:

Statement (2) is correct- India‘s Strategic Forces Command successfully test-fired Prithvi-II missile during night time as part of a user trial by the Army from a test range in Odisha. It is developed by Defence Research and Development Organisation (DRDO) of India under the Integrated Guided Missile Development Program.

Statement (1) is incorrect- Prithvi-II is an indigenously developed nuclear capable missile. The missile is capable of carrying 500 to 1,000 kg of warheads. It is a surface- to-surface missile with a strike range of 350 km.

Recently, it was launched from a mobile launcher from launch complex-3 of the Integrated Test Range (ITR) at Chandipur near Balasore. It was previously successfully test-fired this year during night time from the ITR at Chandipur.

Source: http://vajiramias.com/current-affairs/prithvi- ii/5c19cc952099377306691451/

Q89. Chandra X-Ray Observatory, which was in news recently, is launched by which of the following organisations? a) Indian Space Research Organisation (ISRO) b) National Aeronautics and Space Administration (NASA) c) European space Agency d) China National Space Administration (CNSA)

Answer: b

Explanation:

Due to a technical glitch, The Chandra X-Ray Observatory has entered a protective ‗safe mode‘, which interrupts scientific observations and puts the spacecraft into a stable configuration. NASA's Chandra X-ray Observatory is a telescope specially designed to detect X-ray emission from very hot regions of the Universe such as exploded stars, clusters of galaxies, and matter around black holes.

Because X-rays are absorbed by Earth's atmosphere, Chandra must orbit above it, up to an altitude of 139,000 km (86,500 mi) in space. The Smithsonian's Astrophysical Observatory in Cambridge, MA, hosts the Chandra X-ray Centre which operates the satellite, processes the data, and distributes it to scientists around the world for analysis. It was launched in 1999 and is named after the Nobel Prize-winning

Indian-American astrophysicist Subrahmanyan Chandrasekhar. Hence, option b) is the correct answer.

Source: http://vajiramias.com/current-affairs/chandra- telescope/5c1a02d12099377308fffa67/

Q90. Consider the following kinds of organisms:

(1) Bats

(2) Wasps

(3) Moths

Which of the above is/are pollinating agent/agents? a) 2 only b) 1 and 2 only c) 1 and 3 only d) 1, 2 and 3

Answer: d

Explanation:

Pollination, an important step in the reproduction of seed plants, is the transfer of pollen grains (male gametes) from the male reproductive organ to the female reproductive organ that contains the ovule (female gamete) or transfers it to the ovule itself. Plants, being immobile, normally require agents for the transport of pollen, which are commonly wind, insects, birds, mammals (bats, rodents, primates), and water.

A recent study by scientists of Zoological Survey of India (ZSI) has revealed that Moths are pollinators to a number of flowering plants in the Himalayan ecosystem. Under the project titled ―Assessment of Moths (Lepidoptera) As Significant Pollinators in the Himalayan Ecosystem of North Eastern India‖, scientists of Zoological Survey of India (ZSI) collected moth samples from different ecosystem. The study was carried out in states such as Arunachal Pradesh, and West Bengal. The analysis of proboscis, a long and thread-like organ used to suck flower sap, of a dozen moth species‘ revealed the presence of pollen grains of several flowering plants. On observing the proboscis under scanning electron microscope, it was revealed that these structures are not only meant for sap sucking, but are morphological designed for pollination. The study was unique, as scientist are looking at a new group of

insects (moths) as pollinators. Usually bees, wasps and butterflies are considered as prominent pollinators.

Moths comprise a group of insects related to butterflies, belonging to the order Lepidoptera. According to Researchers, almost two-thirds of common large moth species have declined over the last 40 years in some parts of world mainly due to light pollution (an increase in artificial light in moth habitats). Hence, option d) is the correct answer.

Source: Vajiram Current Affairs- October 29, 2018.

Q91. Consider the following:

(1) Mithun

(2) Eurasian otter

(3) Amur falcons

Which of the above are naturally found in India? a) 1 and 2 only b) 2 and 3 only c) 1 and 3 only d) 1, 2 and 3

Answer: a

Explanation:

Point (1) is correct: The killing of at least 10 mithuns in Assam has created tensions between Arunachal Pradesh and neighbouring Assam. The mithun or gayal (Bos frontalis), is considered a descendant of the Indian Gaur or bison. It is distributed in Northeast India, Bangladesh, northern Myanmar and in Yunnan, China. Reared under free-range conditions in hilly forests, the mithun is known as the ‗cattle of the mountain‘. It plays an important role in the socio-economic and cultural life of tribes such as the Nyishi, Apatani, Galo and Adi in Arunachal Pradesh. The gayal is the state animal of Arunachal Pradesh and Nagaland. They are listed as Vulnerable on IUCN Red List of Threatened species.

Point (2) is correct: Recently Scientists have confirmed the presence of the Eurasian otter in the Western Ghats. Eurasian otter is one of the least-known of India‘s three otter species (the other two being smooth-coated otter and small-clawed

otter). While the species is widespread across Europe, northern Africa and several south Asian countries, it is not as frequently sighted in India. Though protected by the Wildlife Protection Act (1972), otters are often illegally poached for pelts. Recently, scientists affiliated with the Laboratory for Conservation of Endangered Species at Hyderabad‘s Centre for Cellular and Molecular Biology have confirmed the presence of Eurasian otter in Western Ghats. Though the Eurasian otter has been recorded historically from the Western Ghats (Coorg in Karnataka and Tamil Nadu‘s Nilgiri and Palani hill ranges), this is the first photographic and genetic confirmation of its presence here.

Point (3) is incorrect: Since 2010, Umru village on the Assam-Meghalaya border is witnessing a flock of Amur falcons – the world‘s longest travelling raptors – every winters. They are small raptors of the falcon family. Raptors are birds that hunt invertebrates and vertebrates including other birds. Their diet consists mainly of insects, such as termites. They have red feet. They breed in south-eastern Siberia and Northern China, wintering in Southern Africa. They start their journey from South – eastern Siberia and northern China. Then en-route, in the month of October to November, a large number of Amur falcons arrive in the northeast India, especially near the Doyang reservoir in Wokha district of Nagaland in Nagaland for roosting (sleeping/resting). On their return flight they fly over Bangladesh and Myanmar after entering India but skip Nagaland. They are protected under the Wildlife (Protection) Act, 1972. The Nagaland government hosted the first Amur Falcon Conservation Week from November 8 to 10. They are listed as Least Concern on IUCN Red List of Threatened species. Hence, option a) is the correct answer.

Source: http://vajiramias.com/current-affairs/eurasian- otter/5c19cdae2099377308fffa0c/ http://vajiramias.com/current-affairs/mithun/5c1a02112099377308fffa65/

Amur Falcons- Vajiram Current Affairs- October 7, 2018.

Q92. Which one of the following is the best description of the term ‗Principal of non-refoulement‘ that was in the news recently? a) Refugees should not be returned or expelled pending final determination of their status. b) Document about human rights in the areas of sexual orientation and gender identity. c) Prisoners of war shall be released and repatriated without delay after the cessation of active hostilities.

d) It set out six main criteria that National Human Rights Institutions (NHRIs) require to meet.

Answer: a

Explanation:

United Nations expert urged the Government of India to abide by the international norm of non-refoulement and protect the rights of asylum seekers and refugees including Rohingyas. The UN defines refoulement as ―the expulsion of persons who have the right to be recognised as refugees‖. The principle of non-refoulement is set out under the 1951 Convention Relating to the Status of Refugees and its 1967 Protocol (India is not a signatory). The principle is that refugees should not be returned or expelled pending final determination of their status. It is described in Article 33 of the 1951 Convention, also binding on states party to the 1967 Protocol. Hence, option a) is the correct answer.

Source: http://vajiramias.com/current-affairs/non- refoulement/5c19d1e8209937730669145e/

Q93. Ghost net, often mentioned in the news, is related to which of the following? a) Space b) Fishing c) Glaciers d) Air Pollution

Answer: b

Explanation:

The Government of India is in the process of preparing a national ghost net management policy to contain a growing threat to marine ecosystems. Ghost nets are classified under Abandoned, Lost or otherwise Discarded Fishing Gear (ALDFG), which includes lines, traps, hooks, dredges and buoys. Ghost nets can kill marine wildlife, including vulnerable species, and destroy the ecosystems that exist at the lowest level of a body of water. The problem has been worsening with the global expansion in fishing operations. The UNEP, in a 2009 study, states that almost 10 % of all fishing gear is lost or discarded in our oceans annually, in bad weather or when nets get stuck to the rocky bottom. A 2010 Marine Fisheries Census by the Central Marine Fisheries Research Institute (CMFRI) said there were about 1,30,000 gillnets and drift nets in operation in India. Hence, option b) is the correct answer.

Source: http://vajiramias.com/current-affairs/ghost- nets/5c19cd5b2099377308fffa0a/

Q94. Intermediate-range Nuclear Forces (INF) Treaty, often mentioned in the news, is between USA and which one of the following countries? a) North Korea b) China c) India d) Russia

Answer: d

Explanation:

U.S. President Donald Trump confirmed that the U.S. would pull out of the Intermediate-range Nuclear Forces (INF) Treaty with Russia. Intermediate-range Nuclear Forces (INF) Treaty is a crucial Cold War-era treaty banning the development, testing and possession of short and medium range ground- launched nuclear missiles with a range of 500-5,000 km. The treaty, signed in 1987, was central to ending the arms race between the two superpowers, and protected America‘s NATO allies in Europe from Soviet missile attacks. On October 20, 2018, the United States declared its intention to withdraw from the treaty on the ground that Russia has been violating it for many years. Hence, option d) is the correct answer.

Source: http://vajiramias.com/current-affairs/intermediate-range-nuclear-forces- inf-treaty/5c1b2fd820993706fc3da05b/

Q95. ‗Green Good Deeds movement‘ sometimes mentioned in the news, is an initiative of which of the following? a) Union Ministry of Environment, Forest and Climate Change b) United Nations Environment Programme (UNEP) c) World Wildlife Fund (WWF) d) United Nations Educational, Scientific and Cultural Organization (UNESCO)

Answer: a

Explanation:

―Green Good Deeds‖ movement is an initiative of Union Ministry of Environment, Forest and Climate Change. ―Green Good Deeds‖ movement urges people to perform at least one Green Good Deed every day. Some of the 500 Green Good Deeds listed are – planting trees, saving energy, conserving water, use of public transport and promoting carpool etc.

Ministry of Environment launched the ―Harit Diwali-Swasth Diwali‖ campaign to celebrate Diwali in an eco-friendly manner. This campaign was initiated in 2017-18 in northern India wherein large number of school children were advised to celebrate Diwali in an eco-friendly manner by taking pledge to minimize bursting of crackers and gifting plant sapling to their relatives and friends along with sweets. The above campaign was extremely successful and the air quality had not deteriorated post Diwali in 2017 unlike what was experienced in 2016. On the above lines, the Ministry has initiated the similar campaign, but this year the campaign has been extended Pan-India. The ―Harit Diwali- Swasth Diwali‖ campaign is now merged with ―Green Good Deed‖ movement that has been initiated as a social mobilization for conservation and protection of environment. Hence, option a) is the correct answer.

Source: http://vajiramias.com/current-affairs/harit-diwali-swasth- diwali/5c1b3c1e2099370b79fd133f/ Q96. Consider the following Islands:

(1) Agalega

(2) Migingo

(3) Assumption

Which of the above Islands are located in Indian Ocean? a) 1 and 2 only b) 2 and 3 only c) 1 and 3 only d) 1, 2 and 3

Answer: c

Explanation:

Point (1) is correct: Recently, Prime Minister of Mauritius faced tough questions in the Parliament over Indian involvement in the Agalega islands project on the issue of costs and whether it would involve a military component. Agalega archipelago consists of two sparsely populated islands located about 965 km north of mainland Mauritius

in Indian Ocean. It has a population of around 300. India and Agalega archipelago project: Under this, India agreed to provide approximately $87 million to build an airport terminal, extend its runway and refurbish jetties.

Point (2) is incorrect: For over a decade, Migingo Island has been a source of tension between Uganda and Kenya. Migingo is a 2,000-square-metre (0.49-) island in Lake . It is extremely densely populated. The island is the center of a territorial dispute between Kenya and Uganda. Presently, the island is co- managed by both countries. While fishing communities around Lake Victoria have seen their catches slowly diminish over the years, the deep waters surrounding Migingo abound with catch such as Nile perch. This is one of the main reason for both sides claiming the island.

Point (3) is correct: Assumption Island is a small island in the Outer Islands of Seychelles north of Madagascar. In 2018, Seychelles and India signed an agreement to build and operate a joint military facility on a portion of the island.

Source: http://vajiramias.com/current-affairs/agalega- islands/5c1b6e9e2099370b79fd1489/ http://vajiramias.com/current-affairs/migingo-island/5c1b38d92099370b77a8948b/

Q97. With reference to the Financial Stability and Development Council (FSDC), consider the following statements:

(1) The FSDC is headed by the Union Finance Minister.

(2) Its objective is to strengthen and institutionalize the mechanism for maintaining financial stability and enhance inter-regulatory coordination.

Which of the statements given above is/are correct? a) 1 only b) 2 only c) Both 1 and 2 d) Neither 1 nor 2 Answer: c

Explanation:

19th Meeting of the Financial Stability and Development Council (FSDC) was held under the Chairmanship of the Union Finance Minister Arun Jaitley. It reviewed the current global and domestic economic situation and financial sector performance. It was setup in 2010 by Indian Government as an executive body. It replaced the High Level Coordination Committee on Financial Markets (HLCCFM). Hence statement (1) is correct.

Its objective is to strengthen and institutionalize the mechanism for maintaining financial stability and enhance inter-regulatory coordination. Hence statement (2) is correct.

A sub-committee of FSDC has also been set up under the chairmanship of Governor RBI. It discusses and decides on a range of issues relating to financial sector development and stability including substantive issues relating to inter-regulatory coordination.

Source: http://vajiramias.com/current-affairs/financial-stability-and-development- council-fsdc/5c1b828020993711119c09bd/

Q98. With reference to the ‗Graded Response Action Plan‘, sometimes seen in the news, consider the following statements:

(1) It is a Delhi specific comprehensive action plan, prepared by the Central Pollution Control Board (CPCB).

(2) The Union Government has assigned the task of implementation of the Graded Response Action Plan to the Central Pollution Control Board (CPCB).

Which of the statements given above is/are correct? a) 1 only b) 2 only c) Both 1 and 2 d) Neither 1 nor 2 Answer: a

Explanation:

In pursuant to the Supreme Court‘s order of December, 2016 Union government notified a GRAP for Improving air quality in National Capital Region of Delhi in January 2017.

Statement (1) is correct- The Delhi specific comprehensive action plan was prepared by the Central Pollution Control Board (CPCB). The Union Environment Ministry notified a ‗Graded Response Action Plan‘ against air pollution for Delhi and the National Capital Region.

Statement (2) is incorrect- For enforcement of the action plan, the Union Government has assigned the task of implementation of the Graded Response Action Plan to the EPCA (Environment Pollution Control Authority) in pursuance of sub-section (1) of section 3 of the Environment (Protection) Act, 1986.

Warning of a dip in air quality from November 1 to 10, an expert panel, constituted under the Graded Response Action Plan (GRAP) for Delhi, has recommended a ban on construction activities and shutting down of coal and biomass industries.

Source: http://vajiramias.com/current-affairs/graded-response-action-plan- grap/5c1b6b782099370b77a895b8/

Q99. Which of the following straits connects Java Sea to the South China Sea? a) Karimata Strait b) Palk Strait

c) Formosa Strait d) Yucatan Strait

Answer: a

Explanation:

The Java Sea lies between the Indonesian islands of Borneo to the north, Java to the south, Sumatra to the west, and Sulawesi to the east. Karimata Strait to its northwest links it to the South China Sea. It is an extensive shallow sea on the Sunda Shelf. Recently, an Indonesian Lion Air jet crashed into the Java Sea shortly after taking off from Jakarta. Hence, option a) is the correct answer.

Source: http://vajiramias.com/current-affairs/java- sea/5c1b7a91209937103b215f93/

Q100. The only fighter aircraft repair depot of the Indian Air Force (IAF) is located at which of the following places? a) Hindon, Uttar Pradesh b) Ambala, c) , Punjab d) Ojhar, Maharashtra

Answer: d

Explanation:

11 Base Repair Depot (BRD), Ojhar of the Indian Air Force (IAF) handed over the first overhauled Su-30 MKI aircraft to the operational squadron. 11 BRD, Ojhar in Nashik, Maharashtra is the only fighter aircraft repair depot of the Indian Air Force (IAF) and undertakes repair and overhaul of frontline fighters such as MIG-29 and Sukhoi 30 MKI. It was established in 1974 and subsequently renamed as 11 Base Repair depot in 1975.Presently, the upgradation of MIG-29 and repair & overhaul (ROH) of the su-30 MKI aircraft is being undertaken at the depot. Hence, option d) is the correct answer.

Source: http://vajiramias.com/current-affairs/11-base-repair-depot-brd- ojhar/5c1b6ad92099370b76395aaa/